4120 1387
Exam Time
-
NOTEPAD
Results
of 200 questions answered correctly

You have reached of 200 points, ( %)

Your time

Question 1 of 200

1. Examination of a 13-year-old girl revealed acute glomerulonephritis, nephritic syndrome at the initial stage without renal dysfunction. What is the main drug of choice for the basic therapy of this patient?

Explanation

 

Glomerulonephritis  is an immunologically mediated diffuse inflammatory disease which involves both kidneys symmetrically affecting mainly the glomerulus and associated with changes in tubules, interstitial tissue and vessels. The main etiological factor is an infection by group A beta hemolytic streptococcus. note that the question states that this disease is in its initial stage ‘without renal dysfunction’;  this implies that for the basic therapy, an antibiotic ( targeted at the causative agent) should be prescribed for the patient.  In a case where renal function has been affected, other approaches such as dialysis, transplant etc should be considered.

  1.  

2. A 28-year-old patient complains of infertility. The patient has been married for 4 years, has regular sexual life and does not use contraceptives but has never got pregnant. Examination revealed normal state of the genitals, tubal patency. Basal body temperature recorded over the course of 3 consecutive menstrual cycles appeared to have a single phase. What is the most likely cause of infertility?

Explanation

3. A 40-year-old female patient has been hospitalized for attacks of asphyxia, cough with phlegm. She has a 4-year history of the disease. The first attack of asphyxia occurred during her stay in the countryside. Further attacks occurred while cleaning the room. After 3 days of inpatient treatment the patient’s condition has significantly improved. What is the most likely etiological factor?

Explanation

The patient was admitted as a result of an allergic attack. From anamnesis, we find that these attacks mostly occurred while cleaning the room indicating that the causative factor is a household allergen.
4. A 13-year-old girl complains of periodic prickly pain in the heart region. Percussion revealed no changes of cardiac borders. Auscultation revealed arrhythmic enhanced heart sounds, extrasystole at the 20-25 cardiac impulse. ECG showed the sinus rhythm, impaired repolarization, single supraventricular extrasystoles at rest. What is the most likely diagnosis?

Explanation

Vegetative-vascular dysfunction is also referred to as Somatoform autonomic dysfunction. This  is a clinical syndrome that includes disorders of human systems and organs with various origin and manifestations. The vegetative (autonomous) nervous system that has two main branches, the sympathetic branch and the parasympathetic system, provides innervation of internal organs. These systems produce an opposite effect on a human body. Normal systems are balanced, whereas the somatoform autonomic dysfunction disturbs the balance and the effect of either sympathetic or parasympathetic system predominates.

 

In the case of rheumatism, the heart valves will be affected with an underlying streptococcal infection. The appearance of no changes in cardiac borders on percussion rules out the option of carditis.

5. A 28-year-old patient has been taken to a hospital for acute pain in the lower abdomen. There was a brief syncope. The delay ofmenstruation is 2 months. Objectively: the patient has pale skin, AP- 90/50 mm Hg, Ps- 110/min. Lower abdomen is extremely painful. Vaginal examination reveals uterus enlargement. There is positive Promtov’s sign. Right appendages are enlarged and very painful. Posterior vault hangs over. What is the most likely diagnosis?

Explanation

An ectopic pregnancy occurs when implantation takes place outside the walls of the uterus; in most cases it occurs in the fallopian tubes ( tubal pregnancy). The patient presents with a history of missed periods for about 2 months ( an early sign of pregnancy). Typical signs include pelvic and severe lower abdominal pain, mass present in the uterine appendage ( adnexa). Notice that on examination, the patient presents with enlarged and very painful uterine appendages on the right side which confirms a right-sided tubal pregnancy. Right ovary apoplexy refers to the rupture of the ovary, it presents with severe pain, bleeding into the abdominal cavity and signs of irritation of the peritoneum.

 

The inflammation of the uterine appendage, particularly on the fallopian tubes is the disease called Salpingo-Oophoritis. It develops when staphylococci, streptococci, colon bacilli, gonococci, tubercle bacilli or other pathogens enter into fallopian tubes and the ovaries.

6. 5 weeks after hypothermia a 22-year-old patient developed fever, weakness, muscle pain, inability to move independently. Objectively: tenderness, induration of shoulder and shin muscles, restricted active movements, erythema on the anterior surface of chest. There is a periorbital edema with heliotropic erythema. Gottron’s sign is present. What study is required to verify the diagnosis?

Explanation

Gottron’s sign refers to erythematous macules/papules present on the elbows ( sometimes also on the knees) and are specific signs in diagnosis of dermatomyositis.

 

Dermatomyositis is a muscle disease, a long term inflammatory myopathy (muscle disorder) characterised by progressive proximal muscle weakness, Gottron papules and heliotrope rash. For diagnosis, a muscle biopsy is obtained and observed under the microscope; signs such as mononuclear antibodies, abnormal cell degeneration and regeneration  etc are observed.

7. A 24-year-old patient consulted a doctor about enlarged submandibular lymph nodes. Objectively: submandibular, axillary and inguinal lymph nodes are enlarged. Chest radiograph shows enlarged mediastinal lymph nodes. In blood: RBCs - 3, 4 · 1012/l, Hb- 100 g/l, colour index - 0,88, thrombocytes - 190 ·109/l, WBCs - 7, 5 · 109/l, eosinophils - 8%, stab neutrophiles - 2%, segmented neutrophiles - 67%, lymphocytes - 23%, ESR - 22 mm/h. What study is required to verify the cause of lymphadenopathy?

Explanation

8. A patient operated for acute paraproctitis undergoes antibacterial and detoxification therapy, the local course of the disease has the positive dynamics. Since the operation the patient has had chills, pyrexia, tachycardia, euphoria for five days. The doctor suspected sepsis. What study will confirm the diagnosis?

Explanation

 

Sepsis refers to the body’s extreme response to infection. This patient most probably got infected during the surgical procedure; pyrexia, chills, tachycardia etc are key signs of the body’s reaction. To confirm the diagnosis and also find out the particular pathogen, a blood culture should be conducted.

9. A painter working at a motorcar plant has been diagnosed with moderately severe intoxication with amide compounds of benzene. The inpatient treatment resulted in a considerable health improvement. What expert decision should be made in this case?

Explanation

10. A 58-year-old female patient came to the antenatal clinic complaining of bloody light red discharges from the genital tracts. Menopause is 12 years. Gynaecological examination revealed age involution of externalia and vagina; uterine cervix was unchanged, there were scant bloody discharges from uterine cervix, uterus was of normal size; uterine appendages were not palpable; parametria were free. What is the most likely diagnosis?

Explanation

11. A 7-year-old boy had complained of headache, nausea, fatigue for 3 weeks. His condition gradually deteriorated, headache and general weakness progressed. The boy had bronchitis at the age of 3. His father has a history of pulmonary tuberculosis. Objectively: body temperature 37, 5oC, conscious, lies supine, with the hip and knee flexed to 90 degrees, nuchal rigidity +6 cm, partial ptosis of the right eyelid, the dilated right pupil. General hyperalgesia is present. Liquor: transparent, pressure - 400 mm of water column, protein -1,5%, cytosis - 610/3 with predominant lymphocytes, sugar - 1,22 mmol/l, chlorides - 500 mmol/l. What is the most likely diagnosis?

Explanation

Meningitis is an acute infectious disease with involvement of the arachnoid and pia of the brain and spinal cord by pathogenic microorganisms. Etiologically, the various forms include; bacterial, viral. Fungal and tuberculous meningitis. The tuberculous form occurs as a secondary infection; The CSF is under increased pressure, is clear or slightly cloudy, and contains a predominance of mononuclear cells (usually >400/mm3), increased protein (100 to 400 mg/dL), and a decreased glucose content.
12. A 13-year-old girl has a 5-year history of pain in the right hypochondrium irradiating to the right shoulder blade. The pain attacks are usually associated with diet violations, they are short and can be easily relieved by antispasmodic drugs. During a pain attack, palpation of the abdomen is painful, the pain is most intensive in the projection of the gallbladder. What is the most likely diagnosis?

Explanation

13. A 24-year-old patient got a puncture injury below the Poupart’s ligament accompanied by intense arterial bleeding. The best method to temporarily stop the bleeding in the patient would be:

Explanation

 

Poupart ligament refers to the inguinal or groin ligament. Compression bands are also known as pressure bandages;  are used to control bleeding and encourage blood clotting without constricting normal blood circulation. Esmarch tourniquet is used to stop bleeding located on the limbs.

14. A 64-year-old patient complains of severe pain in the right side of chest, dyspnea, dry cough which appeared suddenly on exertion. Objectively: the right side of the chest lags behind in the act of breathing. Percussion reveals tympanic sound. Auscultation reveals pronouncedly diminished breath sounds on the right. Ps- 100/min, weak, arrhythmic. AP- 100/50mm Hg. Cardiac sounds are decreased. What disease can be suspected in this patient?

Explanation

Pneumothorax is air in the pleural cavity i.e., accumulation of air between the lungs and chest wall. Accumulation of air will prevent chest excursion on the affected side during respiration, that is why we see no chest movement on that side ( here the right side of the chest lags during respiration).Pneumothorax develops when air enters the pleural space as the result of disease or injury. This leads to a loss of negative pressure between the two pleural membranes, which can result in the partial or complete collapse of the lung. Pneumothorax is classified as spontaneous or traumatic. Spontaneous pneumothorax can be further classified as primary (i.e., no underlying lung disease) or secondary (i.e., due to underlying lung disease). Any type of pneumothorax can progress to tension pneumothorax. Symptoms include;  dyspnea, ipsilateral chest pain, diminished breath sounds, and hyper-resonant percussion on the affected side

 

Hemothorax, hydrothorax will give dull sounds during percussion because of the presence of fluid.

15. An 18-year-old patient since childhood suffers from bleeding disorder after minor injuries. His younger brother also has bleeding disorders with occasional haemarthrosis. Which laboratory test will be informative for diagnosis verification?

Explanation

Clotting time is the time required for a sample of blood to coagulate in vitro under standard conditions. The time taken for blood to clot mainly reflects the time required for the generation of thrombin.  If the plasma concentration of prothrombin or of some of the other factors is low (or if the factor is absent, or functionally inactive), clotting time will be prolonged. The expected range for clotting time is 4-10 mins.

 

Clotting time involves the measurement of prothrombin time (PT) and activated partial thromboplastin time (aPTT). Prothrombin time assesses the extrinsic (factor VII) and common (factors X, V, II [prothrombin] and I [fibrinogen]) pathways, whereas aPTT assesses the intrinsic (prekallikrein, HMWK, and factors XII, XI, IX, and VIII) and common pathways.

16. A patient complains of fatigue, lack of appetite, pain and burning sensation in the tongue, numbness of the distal limbs, diarrhea. Objectively: pale skin with lemon-yellow tint, face puffiness, brown pigmentation in the form of a \\\"butterfly\\\", bright red areas on the tongue. The liver is 3 cm below the costal margin, soft. Blood count: RBCs - 1, 5 · 1012/l, colour index - 1,2, WBCs - 3, 8 · 109/l, thrombocytes - 180 · 109/l, eosinophils -0%, stab neutrophils - 1%, segmented neutrophils - 58%, lymphocytes - 38% monocytes - 3%, RBC macrocytosis. ESR - 28 mm/h. What diagnosis are these presentation typical for?

Explanation

From the blood analysis given, Macrocytes are present- this refers to irregular large Red Blood cells. Also the color index is elevated. The erythrocyte level is also very low ( norm- Female: 3.5 − 5.5 · 1012/L) - this indicates an anemia . The major causes of macrocytic anemia include Vitamin B12 deficiency, Vitamin B9 (Folic Acid) deficiency or medications such as antiretroviral drugs. 

Symptoms include loss of appetite, brittle nails , pale skin, fatigue etc. To differentiate between B12 or Folic acid deficiency, Vit B12 deficiency is often associated with nervous system symptoms like paresthesias - subacute combined degeneration due to abnormal myelin synthesis. These nervous system symptoms are absent in Folic Acid Deficiencies.

With the symptoms listed in the question, and the presence of anemia and macrocytes - it is safe to say Vitamin B12 deficiency is the best choice.

 

Ascorbic acid or vitamin C deficiency will lead to scurvy, petechial bleeding or bruises are common in this case. Vitamin B1 deficiency will lead to the development of Beri-Beri.

17. Medical examination of a 19-year-old worker revealed generalized lymphadenopathy mainly affecting the posterior cervical, axillary and ulnar lymph nodes. There are multiple injection marks on the elbow bend skin. The man denies taking drugs, the presence of injection marks ascribes to influenza treatment. Blood count: RBCs- 3, 2·1012/l, Hb- 100 g/l, WBCs- 3, 1 · 109/l, moderate lymphopenia. What study is required in the first place?

Explanation

From physical examination, we observe the presence of multiple injection marks on the patient’s body. This should raise a suspicion about an intravenous drug abuser and an infection with HIV. HIV is an immunosuppressive disease and patients suffering from this disease are prone to having opportunistic infections. For diagnosis of HIV infection, HIV antibodies are found in serum or saliva via ELISA test ( Enzyme linked Immunosorbent assay) and is usually confirmed by a western blot test.
18. A patient got flame burns of both hands. On the dorsal and palmar surface of hands the blisters are filled with serous fluid. The wrist joint region is hyperemic. The forearms were not injured. What is the provisional diagnosis?

Explanation

rule of “nines” – area of different areas of the body is proportional: anterior surface of the trunk – 18 %, posterior – 18 %, lower limb – 18 %, external genitals – 1 %);

   rule of  “palm”. It is used if burns are limited and located on different areas of the body. According to the rule, the palm takes 1 % of the skin surface.

Division of the burns on superficial (I, II, IIIA st.) and deep (IIIB-IV st.)

I stage – hyperemia of the skin

II stage – separation of epidermis with formation of bullas/

III A stage – necrosis of superficial layers of the skin with saving of bulbs hair, sweat glands and sebaceous glands.

IIIB stage – necrosis of all the derma

 

IV stage – necrosis of the skin and underlying tissues.

19. During the periodic medical examination an assembly fitter (works on soldering details) didn’t report any health problems. Closer examination revealed signs of asthenic vegetative syndrome. Blood included red blood cells with basophilic aggregations and a somewhat higher number of reticulocytes, urine had a high concentration of delta aminolevulinic acid. The complex of symptoms indicates the initial stage of chronic intoxication with:

Explanation

 

Delta-aminolevulinic acid dehydratase is an enzyme necessary for the synthesis of heme. In lead poisoning, this enzyme is inhibited leading to anemia and an increased delta aminolevulinic acid content in urine. N/B a high reticulocyte number means an increase in RBC production ( new cells) and is observed in cases such as bleeding, high altitude or anemia. The above patient presents with signs of asthenic vegetative syndrome coupled with the above stated reasons confirming a case of lead poisoning.

20. A patient complains about sudden onsets of paroxysmal pain in the right lumbar region. 2 hours after the onset the patient had hematuria. Plain radiograph of the lumbar region shows no pathological shadows. USI reveals pyelocaliectasis on the right, the left kidney is normal. What is the most likely diagnosis?

Explanation

Notice that on ultrasound examination, pyelocaliectasis was observed. This refers to the dilation of the renal pelvis and calyces. The most common causes of pyelectasis are:

  • Ureteropelvic junction obstruction: A blockage of urine between the kidneys and the ureter

  • Vesicoureteral reflux: An abnormal flow of urine from the bladder to the kidneys.

This Presents with excruciating spasms around the lumbar region ( renal colic).
21. Examination of a group of persons living on the same territory revealed the following common symptoms: dark yellow pigmentation of the tooth enamel, diffuse osteoporosis of bone apparatus, ossification of ligaments and joints, functional disorders of the central nervous system. This condition may be caused by the excessive concentration of the following microelement in food or drinking water:

Explanation

 

Presence of dark-yellow pigmentation of the tooth enamel, diffuse osteoporosis of bone apparatus, ossification of ligaments and joints are specific symptoms that indicate Fluorosis. This is a condition that occurs as a result of excessive intake of fluorine

22. A 50-year-old patient complains of bursting pain in the left lower limb that is getting worse on exertion, swelling in the region of shin and foot. Objectively: left shin and foot are doughy, skin of the lower shin is indurated and has a bronze tint, subcutaneous veins are dilated, there is an ulcer with necrotic masses. What is the most likely diagnosis?

Explanation

Post-thrombotic syndrome refers to symptoms and signs of chronic venous insufficiency that develop following deep vein thrombosis (DVT) and is a common, burdensome, and costly complication . The term "post-thrombotic" replaces the prior terminology "postphlebitic" syndrome. A combination of reflux due to valvular incompetence and venous hypertension due to thrombotic obstruction is thought to underlie the pathophysiology of post-thrombotic syndrome . Symptoms and signs of post-thrombotic syndrome may include leg pain, leg heaviness, vein dilation, edema, skin pigmentation, and venous ulcers.

Deep vein thrombosis occurs when a blood clot  forms in a deep vein usually in the thigh and lower leg. It presents with pain, warmth and tenderness of the affected area. 

 

Acute arterial thrombosis occurs after an endovascular procedure results of the erosion or rupture of atherosclerotic plaque and/or distal embolization (DE). Both occurrences activate the coagulation and platelet systems, resulting in the occlusion of the artery by a thrombus.

23. A 12-year-old boy has a 6-year history of insulin dependent diabetes. The disease is labile. Since recently there have been periodical rises in blood pressure. Microalbuminuria test gave positive results. The patient’s condition corresponds with the following stage of diabetic nephropathy:

Explanation

 In Diabetes Mellitus, Mogensen identified 5 stages of renal dysfunction:

 
  • Stage 1: Renal hypertrophy and hyperfiltration – glomerular filtration rate may be increased 20 - 40% 

  • Stage 2: Is clinically ‘silent’ but hyperfiltration persists and is correlated with mild hyperglycemia 

  • Stage 3: Microalbuminuria is present 

  • Stage 4: Overt nephropathy with proteinuria, hypertension and progressive renal failure 

  • Stage 5: End-stage renal failure develops

24. A 48-year-old patient was found to have diffuse enlargement of the thyroid gland, exophthalmia, weight loss of 4 kg in 2 months, sweating. Objectively: HR- 105/min, AP- 140/70 mm Hg. Defecation act is normal. What kind of therapy is recommended in this case?

Explanation

 

The patient experiences an enlarged thyroid gland, tachycardia, loss of weight  and exophthalmos which are symptoms of Hyperthyroidism ( thyrotoxicosis). For treatment Mercazolil (thiamazole) is administered. Propranolol is a beta blocker and used in heart related issues while thyroxine is indicated for hypothyroidism.

25. A 48-year-old male patient complains of constant pain in the upper abdomen, mostly on the left, that is getting worse after taking meals; diarrhea, weight loss. The patient is an alcohol abuser. 2 years ago he had acute pancreatitis. Blood amylase is 4 g/h·l. Coprogram shows steatorrhea, creatorrhea. Blood glucose is 6,0 mmol/l. What treatment is indicated for this patient?

Explanation

This patient has a case of acute Pancreatitis. Recall that the pancreas has both endocrine and exocrine functions; for the exocrine part, it produces enzymes that aid in digestion such as amylase, lipase, trypsin. Absence of these enzymes will lead to symptoms such as steatorrhea  (fats in feces), creatorrhea (undigested muscles in feces) etc. Panzinorm Forte is a combination of digestive enzymes. These enzymes are normally produced by the pancreas and are important for digesting fats, proteins, and sugars.

Panzinorm Forte is used to replace digestive enzymes when the body does not have enough of its own. Certain medical conditions can cause this lack of enzymes, such as cystic fibrosis, pancreatitis, pancreatic cancer, or pancreas surgery.

Panzinorm Forte may also be used to treat a condition called steatorrhea (loose, fatty stools).

 

Pirenzepine is an M1 selective antagonist used in treatment of peptic ulcer ( it reduces gastric acid secretion and muscle spasm). Contrykal is a protease inhibitor and is used in reducing blood loss in surgical procedures. Drotaverine is an antispasmodic drug used for cervical dilation in childbirth

26. A patient is on the sick leave for 4 months continuously from the date of injury. The treatment is going to last for 1-2 months more. Who has the right to extend the duration of medical certificate for this patient?

Explanation

The right of prolongating a medical certificate (in this case a sick list) of invalidity alongside with the medical treatment by the doctor is granted by:

• Chief medical officer (assistant of the chief medical officer on expertise of ability for work);

• Manager of department;

• Medical-consulting commission also known as medical expertise committee

 

The medical-consulting commission (MCC) is formed in case the doctors staff is above 15 doctors.

27. Full-term pregnancy. Body weight of the pregnant woman is 62 kg. The fetus has the longitudinal position, the fetal head is pressed against the pelvic inlet. Abdominal circumference is 100 cm. Fundal height is 35 cm. What is the approximate weight of the fetus?

Explanation

In recent studies, the value of abdominal girth ( belly circumference) and fundal height ( length from the mother’s uterus to the top of the pubic symphysis) can be used in predicting the weight of a baby. This method is used in order to quickly determine low birth weight babies. It is done by multiplying  the belly circumference by the fundal height; giving you the predicted weight in grams (g), (divide by 1000 for the value in kg). Ie, 100x35= 3500g or 3.5kg.
28. A rural hospital serves 6200 people. Preventive examinations were planned for 560 farm workers with different risk factors. 400 workers underwent preventive examination. 120 individuals were found to have cardiovascular problems, 90 of them were registered with a dispensary department for health care maintenance. Which indicator should be used to assess the organization of health care maintenance at the hospital?

Explanation

29. A 20-year-old patient complains of severe headache, double vision, weakness, fever, irritability. Objectively: body temperature is at the rate of 38, 1oC, the patient is reluctant to contact, sensitive to stimuli. There is ptosis of the left eyelid, exotropia, anisocoria S>D, pronounced meningeal syndrome. On lumbar puncture the cerebrospinal fluid flowed out under a pressure of 300 mm Hg, the fluid is clear, slightly opalescent. 24 hours later there appeared the fibrinous film. Protein - 1,4 g/l, lymphocytes -600/3 per mm3, sugar - 0,3 mmol/l. What is the provisional diagnosis?

Explanation

Meningitis is an acute infectious disease with involvement of the arachnoid and pia of the brain and spinal cord by pathogenic microorganisms. Etiologically, the various forms include; bacterial, viral. Fungal and tuberculous meningitis. The tuberculous form occurs as a secondary infection; The CSF is under increased pressure, is clear or slightly cloudy, and contains a predominance of mononuclear cells (usually >400/mm3), increased protein (100 to 400 mg/dL), and a decreased glucose content.
30. A 32-year-old patient has developed an acute condition after hypothermia: temperature - 40oC, cough with 200ml of sputum per day. The sputum is purulent, foul-smelling. To the right of the lower lobe the mixed moist rales can be auscultated. Blood tst results: WBCs -18, 0 · 109/l, ESR - 45 mm/h. Radiographically: in the lower lobe of the right lung there is a thick-walled cavity up to 6 cm in diameter with a high horizontal level. What is the most likely diagnosis?

Explanation

31. A family consists of 5 persons. The husband is a stope miner. His spouse is a housewife. Their 20-year-old daughter works as a kindergarten teacher. Their 18-year-old son is a student. The grandmother is a pensioner, she has diabetes. Which member of this family can be primarily classed among the group of persons with a high risk of tuberculosis in the planning of preventive examinations for tuberculosis?

Explanation

Generally, persons at high risk for developing TB disease fall into two categories:

  • Persons who have been recently infected with TB bacteria

  • Persons with medical conditions that weaken the immune system

 

Persons in the second group include individuals with the following diseases; HIV infection (the virus that causes AIDS), Substance abuse, Silicosis, Diabetes mellitus, Severe kidney disease, Low body weight, Organ transplants, Head and neck cancer, Medical treatments such as corticosteroids or organ transplant, Specialized treatment for rheumatoid arthritis or Crohn’s disease

32. A newborn’s head is of dolichocephalic shape, that is front-to-back elongated. Examination of the occipital region revealed a labour tumour located in the middle between the prefontanel and posterior fontanel. Specify the type of fetal presentation:

Explanation

33. During medical monitoring of labor and professional training of 6th grade schoolchildren the school doctor found that lessons in carpentry workshop are held on Friday as the fourth and fifth lesson of a total of 90 minutes, the motor density at the first lesson is 69%, and 65% at the second. Does the lesson organization meet the hygienic requirements?

Explanation

34. During coal extraction in a mine the concentration of coal dust in the working area is 450 mg/m3 (MPC is 10 mg/m3). What occupational respiratory disease may develop in miners?

Explanation

Pneumoconiosis is a restrictive lung disease  caused by the inhalation of dust leading to fibrosis. Depending on the type of dust inhaled, different types exist. They include; Siderosis (iron ore), byssinosis (cotton), anthracosis (coal), Asbestosis (asbestos), silicosis (silica dust).the maximum permissible concentration of coal is 10 mg/m3, this patient is in an area where the coal content is  450 mg/m3. Accumulation of coal dust in the lungs will lead to anthracosis.
35. A patient with suspected pheochromocytoma has normal blood pressure in the periods between the atacks and a tendency to tachycardia. Urine test revealed no pathology. It was decided to use a provocative test with histamine. What medication should be prepared to provide emergency care in case of a positive test result?

Explanation

Pheochromocytoma is a tumor that affects the chromaffin cells of the adrenal medulla. It leads to the increased production of catecholamines ( epinephrine and norepinephrine) which have vasoconstrictive effects leading to hypertension. Amongst the listed drugs, Phentolamine which is a non-selective alpha adrenergic receptor blocker is used in the management of hypertensive emergencies majorly due to pheochromocytoma. This drug has a vasodilating  effect. 

 

Pipolphen ( promethazine) is an antihistamine and is used in the treatment of allergic reactions, motion sickness, anaphylaxis etc. Mesaton is a decongestant and a vasopressor.

36. A 54-year-old patient complains of frequent painful urination, chills, fever up to 38oC. Urine test results: protein -0,33 g/L, WBCs - up to 50-60 in the field of vision, RBCs - 5-8 in the field of vision, gram-negative bacilli. Which of the listed antibiotics should be preferred in this case?

Explanation

 

From the results of physical examination and biochemical analysis, we can conclude that this patient is having a urinary tract infection ( UTI) most likely of bacterial origin. From the groups of antibiotics listed below, Ciprofloxacin is the best of choice because of its high bacteriological and clinical cure rates as well as low resistance rate from bacterias that cause UTIs.

37. The share of circulatory diseases was 15% in the total of registered diseases among city population. What statistic indicator is it?

Explanation

The extensive statistical indicator is used to determine a structure of a disease ( in this case, how many percent of the total diseased fall under cardiovascular cases) ie., it shows, what part from the general number of all diseases is made with this or that disease which enters into total.

The intensive parameter characterizes frequency or distribution. It shows how frequently the given phenomenon occurs in the given environment.

 
38. During a regular medical examination at a metallurgical plant 20% of workers were found overweight (body weight was 5-14% higher than normal), and had early signs of obesity (grade I-II) with Quetelet index from 26 to 30. What products share must be reduced in the diet of this group of people in the first place in order to normalize their body weight?

Explanation

 

These factory workers are obesed, with BMI ( quetelet index) - 26-30. In relation to diet, carbohydrate and fatty food groups play a large role in obesity; therefore in terms of diet food materials that contain large contents of these groups should be controlled. This includes bakery products such as flour, wheat etc.

39. Several hours before, a 28-year-old patient suddenly developed acute headache and repeated vomiting, then lost consciousness. Objectively: focal neurological symptoms were not found. Pronounced meningeal symptoms were revealed. AP - 120/80 mm Hg. According to clinical and liquorological findings the patient was diagnosed with subarachnoid haemorrhage. After administration of dehydrants the patient’s condition somewhat improved. What is the main component of further emergency care?

Explanation

The term subarachnoid hemorrhage (SAH) refers to extravasation of blood into the subarachnoid space i.e. the space between the pia and arachnoid membranes.

 

The subarachnoid space is the space where the cerebrospinal fluid (CSF) circulates, therefore, hemorrhage into this space will result in bloody CSF as mentioned in the question or CSF liquor with high content of erythrocytes. The CSF is responsible for protecting your brain from injury by serving as a cushion. A hemorrhage in this space can cause a coma, paralysis, and even death. It is often characterized by a severe headache in adults (often referred to as ‘worst headache of my life’).

From the list of drug groups, anticoagulants ( against coagulation), antiaggregants (against aggregation) and fibrinolytics ( breakdown fibrin) will worsen the current situation if administered. A coagulant will help in the management of the hemorrhagic  situation.

40. Gastric juice analysis of a 42-year-old male patient revealed absence of free hydrochloric acid at all stages. Endoscopy revealed pallor, thinning of gastric mucosa, smoothed folds. Microscopically the atrophy of glands with intestinal metaplasia was found. What disease is this situation typical for?

Explanation

The following types of chronic gastritis include;

Type A is caused by your immune system destroying stomach cells. And it can increase your risk of vitamin deficiencies, anemia, and cancer.

Type B, the most common type, is caused by Helicobacter pylori bacteria, and can cause stomach ulcers, intestinal ulcers, and cancer.

Type C is caused by chemical irritants like nonsteroidal anti-inflammatory drugs (NSAIDs), alcohol, or bile. And it can also cause stomach lining erosion and bleeding.

Observe that on examination, free HCL is absent and results from endoscopy and microscopy indicate the gastric cells have been destroyed - this occurs in Type A gastritis.

 

In menetrier's disease, we see an overgrowth of gastric cells ( foveola) found in the gastric lining which leads to the presence of large gastric folds.

41. A 45-year-old female patient complains of frequent liquid stools with a lot of mucus, pus and blood; pain across the abdomen, loss of 7 kg within 6 months. She has a 1-year history of non-specific ulcerative colitis. What group of drugs should be preferred for this patient?

Explanation

 

This patient is said to have a nonspecific ulcerative colitis which is an inflammatory bowel disease characterised by the presence  of ulcers and sores in the innermost lining of your large intestine (colon) and rectum. On the other had, corticosteroids possess anti-inflammatory, immunosuppressive and anti-pyretic actions, in patients with inflammatory bowel diseases such as chrohns and ulcerative colitis, these drugs are administered as short term treatment because they quickly reduce inflammation ( within few days ).

42. A 24-year-old female teacher complains of dizziness and heart pain irradiating to the left nipple. Pain is not associated with physical activity and cannot be relieved by nitroglycerin, it abates after taking Valocordin and lasts an hour or more. The patient has a nearly 2-year history of this disease. Objectively: Ps- 76 bpm. AP- 110/70 mm Hg. Heart borders are normal, heart sounds are clear. The ECG shows respiratory arrhythmia. Radiograph of the cervicothoracic spine shows no pathology. Lungs, abdomen are unremarkable. What changes in blood formula can be expected?

Explanation

 

From the description of the above patient, we can deduce that the issue is more psychological/mental than physical. This can be proved by the fact that all diagnostic procedures are without any pathological changes. Note that the problem was resolved when the patient was prescribed  Valocordin (corvalol) a tranquilizer indicated for neuroses. Since it is a mental issue, the blood formula will show no changes.

43. A 60-year-old male patient, who works as a construction worker, complains of pain in the right hip and knee joints, that is getting worse on exertion. These presentations have been observed for the last 5 years. Objectively: the patient is overnourished. Right knee joint is moderately deformed. Examination of other organs and systems revealed no pathology. Blood tet results: WBCs -8, 2 · 109/l, ESR - 15 mm/h. Uric acid - 0,35mmol/l. What is the most likely diagnosis?

Explanation

Osteoarthritis is the inflammation of the joint that occurs due to continuous exertion of that area ( wear and tear); unlike Rheumatoid arthritis that is of autoimmune origin.  Observe that the presentation of this patient has been observed for the last 5 years, is one sided and worsens on exertion- these facts help differential from rheumatoid arthritis that has a rapid onset ( weeks- months), occurs symmetrically and improves upon usage of the affected area. 

 

Reactive arthritis is an autoimmune condition that develops in response to an infection in another part of the body. Coming into contact with bacteria and developing an infection can trigger reactive arthritis. It has symptoms similar to various other conditions collectively known as "arthritis,". It is caused by another infection and is thus "reactive". The symptoms of reactive arthritis very often include a combination of three seemingly unlinked symptoms—an inflammatory arthritis of large joints, inflammation of the eyes (conjunctivitis and uveitis), and urethritis. A useful mnemonic is "the patient can't see, can't pee and can't bend the knee" or "the patient can't see, can't pee and can't climb a tree". Also known as Reiter’s syndrome.

44. Chief physician of a polyclinic encharged a district doctor with a task to determine the pathological prevalence of disease N in his district. What document allows to estimate the disease prevalence in the population of a medical district?

Explanation

45. A 32-year-old female complains of dizziness, headache, palpitation, tremor. For the last several months she has been under outpatient observation for the increased arterial pressure. Since recently such attacks have become more frequent and severe. Objectively: skin is covered with clammy sweat, tremor of the extremities is present. HR- 110/min, AP- 220/140 mm Hg. Heart sounds are muffled. Blood test results: WBCs - 9, 8 · 109/l, ESR - 22 mm/h. Blood glucose - 9,8 millimole/l. What disease is the most likely cause of this crisis?

Explanation

The woman not only shows signs of hypertension but also presents with changes relating to body metabolic activities ( tremors, tachycardia, dizziness, palpitations, sweating - these are signs relating to stress ( caused by the release of stress hormones ie, catecholamines). Pheochromocytoma is a tumor of the adrenal gland, it is characterised by the increased production of catecholamines ( adrenaline, noradrenaline) coupled with stress symptoms. Essential hypertension is also called primary hypertension- a form of hypertension with no underlying disease as its cause. Preeclampsia is a pregnancy-associated hypertension coupled with other signs. Primary hyperaldosteronism  or Conn’s disease is also associated with the above stated signs , its other signs may include kidney related problems, excessive urination, low potassium content, flank pain etc.
46. A 55-year-old male has a 1,5-year history of viral cirrhosis with symptoms of portal hypertension. Over the last month the weakness has progrssed, there appeared coffee ground vomit. Fibrogastroduodenoscopy revealed variceal esophageal haemorrhage. What drug should be used to reduce the pressure in the portal vein?

Explanation

Note that vasopressin is the most potent vasoconstrictor of internal organs, it decreases blood flow to all abdominal organs, decreasing venous inflow into the portal veins and portal pressure. Vasopressin always should be accompanied by intravenous nitroglycerin  to maintain a systolic blood pressure of greater than 90 mm Hg.

 

Reserpine is an antihypertensive drug and acts by reducing the amount of norepinephrine. Calcium gluconate is used in the treatment of magnesium toxicity, hypocalcemia and hyperkalemia.

47. In an urban settlement situated on the riverbank an outbreak of hepatitis A was registered. The disease might have water origin. This assumption can be confirmed by growth of the following indicators of water quality:

Explanation

 

Coliphages are microbial indicators specified in the Ground Water Rule (GWR) that can be used to monitor for potential fecal contamination of drinking water. Number of bacteria of group of an intestinal rod (coli-index) in 1000 ml of water - no more than 3.Common microbial number in one ml of water - no more than 100.

48. A 9-month-old child presents with fever, cough, dyspnea. The symptoms appeared 5 days ago after a contact with a person having ARVI. Objectively: the child is in grave condition. Temperature of 38oC, cyanosis of nasolabial triangle is present. RR- 54/min, nasal flaring while breathing. There was percussion dullness on the right below the scapula angle, and tympanic sound over the rest of lungs. Auscultation revealed bilateral fine moist rales predominating on the right. What is the most likely diagnosis?

Explanation

 

The presence of percussion dullness on the right just below the scapula accompanied by the result from auscultation (presence of fine moist crackles) are common signs found in cases of pneumonia. Pneumonia is an inflammation of the pulmonary tissue that occurs as a result of an infection e.g. bacterial, viral etc. laryngotracheitis is also termed Croup; it involves an infection of the larynx, trachea and lungs.

49. A 40-year-old patient complains of fever up to 39oC, cough with sputum and blood admixtures, dyspnea, weakness, herpetic rash on the lips. Objectively: respiration rate - 32/min. Under the shoulder blade on the right the increased vocal fremitus and dullness of percussion sound were revealed. Auscultation revealed bronchial respiration. Blood count: WBCs - 14 · 109/l, ESR - 35 mm/h. What is the provisional diagnosis?

Explanation

50. A patient underwent stomach resection. During the operation, the left upper limb of the patient was abducted and fixed to the operating table for anesthetic management. Postoperatively, the patient developed dysfunction of the upper extremity in form of \\\\\\\"drop hand.\\\\\\\"This symptom results from the damage of the following nerve:

Explanation

[caption id="attachment_1210" align="alignnone" width="300"] .[/caption]

 Ulnar Nerve: Sensory innervation to the Fifth and medial half of the fourth digits; Motor innervation: flexion, adduction and abduction of 4th and 5th digits. Injury to this nerve causes Claw hand

Median Nerve: innervates the skin of the palmar side of the index finger, thumb, middle finger, and half the ring finger, and the nail bed; motor innervation: flexion of lateral fingers, thumb opposition, lumbricals of index and middle fingers. Injured in Carpal Tunnel syndrome and leads to ape’s hand.

Radial Nerve: Sensory innervation to the dorsum (back) of the hand except the little finger (supplied  by ulnar). Motor innervation to the dorsum of the hand. Injury leads to loss of grip strength, wrist drop ‘drop hand’.

51. In the morning a patient had nausea, abdominal discomfort, single vomiting, dry mouth. In the evening, the patient presented with the increasing general weakness, double vision, difficult swallowing of solid food. Objectively: ptosis, mydriasis, anisocoria, absence of gag and pharyngeal reflex, dry mucous membranes. The previous evening the patient had dinner with canned food and alcohol. What is the presumptive diagnosis?

Explanation

 

Botulism is an illness caused by Botulinum toxin , a toxin produced by Clostridium Botulinum ( a gram positive, spore forming, anaerobic bacteria). Foodborne botulism is gotten from eating infected food especially canned food. Symptoms include; disturbed vision (diplopia), flaccid paralysis, dysphagia, dyspnea etc. It is treated with Botulinum antitoxin.

52. An 8-year-old child complains of fever up to 38, 8oC, throat pain when swallowing, skin rash. Objectively: lacunar tonsillitis, circumscribed hyperaemia and enanthema of soft palate, pinoint-sized skin rash, mostly in the folds and on the flexor surfaces of the extremities, pale nasolabial triangle. Which antibiotic should be administered in the first place?

Explanation

Notice that this patient has scarlet fever which is characterised by the presented symptoms; fever, difficulty swallowing, hyperemic soft palate/ tongue, skin rash on flexors and folds, pale nasolabial triangle. This disease is as a result of a Group A streptococcal  or streptococcus pyogenes infection. For its treatment an antibiotic regimen should be followed; the preferred choice of antibiotics is Penicillin or Amoxicillin.
53. A 28-year-old female patient with a six-year history of Raynaud’s syndrome has recently developed pain in the small joints of hands, difficult movement of food down the esophagus. What kind of disease can you think of in this case?

Explanation

Systemic scleroderma  is a chronic multisystem disorder of unknown etiology characterized clinically by thickening of the skin caused by accumulation of connective tissue and by involvement of visceral organs, including the gastrointestinal tract, lungs, heart, and kidneys. Common symptoms include Raynaud phenomenon, polyarthralgia, dysphagia, heartburn, and swelling and eventually skin tightening and contractures of the fingers. 

In Periarteritis nodosa (PAN), small and medium sized vessels are inflamed leading to disruption in major organs such as nerves, intestinal tract, heart, and joints.

 

Systemic lupus erythematosus is characterised by a malar/ butterfly rash.

54. A 16-year-old girl had had polyuria, polydipsia for 2 months. She had lost 8 kg with a good appetite. The patient was urgently hospitalized for abdominal pain and nausea. Examination revealed glycemia at the rate of 18 mmol/l, glycosuria at the rate of 24 g/l. Insulin and infusion of isotonic solutions of sodium chloride and glucose eliminated these problems, including thirst. What is the most likely diagnosis?

Explanation

The key point here is ‘ Insulin and infusion of isotonic solutions of sodium chloride and glucose eliminated these problems’ which confirms this patient has a type 1 diabetes mellitus ( insulin dependent). Recall that type 1 insulin diabetes mellitus is usually noticed at a young age whereas type 2 ( non- insulin dependent) is observed at the latter ages of life as well as in obesed individuals. 

Renal glucosuria is seen when the kidney excretes glucose with urine in individuals with normal or low blood glucose level.

 

Diabetes insipidus is related to the antidiuretic hormone also called vasopressin. In this case, the individual excretes large volumes of urine.

55. A boy was born at 32 weeks gestation. 2 hours after birth he developed respiratory distress. The RD severity assessed by Silverman score was 5. The respiratory disorders progressed, respiratory failure couldn’t be eliminated by Martin-Bouyer CPAP (continuous positive airway pressure). Ro-gram of lungs shows reticular and nodular pattern, air bronhogram. What is the most likely cause of respiratory distress syndrome?

Explanation

56. An 8-year-old child with a 3-year history of diabetes was hospitalized in hyperglycemic coma. Specify the initial dose of insulin to be administered:

Explanation

 

Initial intravenous administration of 10 to 14 units of short-acting insulin has to be prescribed for the patient during the first hour. Continuous intravenous infusion of insulin in a dose 0,1 unit/kg/hour in 0,9 % sodium chloride infusion has to be given after that.

57. A 12-year-old girl undergoes regular gastroenterological check-ups for duodenal ulcer, biliary dyskinesia. What is the recommended frequency of anti relapse treatment?

Explanation

58. A 47-year-old patient came to see a doctor on the 7th day of disease. The disease developed very fast: after the chill body temperature rose up to 40oC and lasted up to 7 hours, then it dropped abruptly, which caused profuse sweat. There were three such attacks occuring once in two days. Two days ago the patient arrived from Africa. Objectively: pale skin, subicteric sclera, significantly enlarged liver and spleen. What is the cause of fever attacks in this disease?

Explanation

From anamnesis, we understand that the patient has just returned from africa , this should prompt the idea of a malarial infection. Malaria is caused by a plasmodium infection (P.vivax, P.falciparum, P.malariae, P.ovale. In its life cycle, various stages are involved ( see image). 

The initial symptoms of malaria infection are nonspecific and can include headache, nausea, vomiting, photophobia and muscle aches. A malarial paroxysm is marked by onset of a sudden shaking chill which may last from 10 to 15 minutes or perhaps longer. Elevated temperature accompanies the paroxysm and may be sustained for typically 10 hours or more. This cycle repeats itself every 36 to 72 hours depending on which species the human host has been infected with. The phase of erythrocytic schizogony takes place within  day 6-15 of the disease ( patient arrives on Day 7); it is characterised by the production of merozoites which are released into the bloodstream.
59. On the 2nd day of illness a 27-year-old patient complains of unbearable headache, repeated vomiting. Objectively: the patient is in a grave condition. He is conscious but adynamic. Lies in a forced position with his head thrown back. There is no skin rash. Nuchal muscles are evidently rigid, there are Kernig’s and Brudzinski’s signs. to- 39, 5oC, Ps- 120/min, AP- 130/80 mm Hg. The leading syndrome of this disease is caused by:

Explanation

60. A 42-year-old female patient suffers from micronodular cryptogenic cirrhosis. Over the last week her condition has deteriorated: she developed convulsions, mental confusion, progressing jaundice. What study may give reasons for such aggravation?

Explanation

Recall that ammonia is converted via the urea or ornithine cycle into urea; this conversion takes place in the mitochondria of liver cells. This patient has liver cirrhosis which means that liver cells are damaged; the presented symptom of convulsions and mental confusion are due to the fact that ammonia can cross the blood brain barrier thereby affecting the brain’s function. To confirm this, the level of ammonia in the patient’s blood should be checked.

 

Alpha fetoprotein is a tumor marker that indicates tumors related to the liver, ovaries, testes etc. In embryology, it is elevated in cases of neonatal defects such as down’s syndrome and spina bifida.

61. A 60-year-old patient complains of recurrent pain in the proximal interphalangeal and wrist joints, their periodic swelling and reddening that have been observed for 4 years. X-ray picture represents changes in form of osteoporosis, joint space narrowing and single usuras. What is the most likely diagnosis?

Explanation

62. A 28-year-old patient has been hospitalized for the pain in the epigastric region. He has a 10-year history of duodenal ulcer (DU). Recently, the pain character has changed: it became permanent, persistent, irradiating to the back. There are general weakness, dizziness, fatigue. The patient has put off weight. Objectively: HR- 68/min, AP- 120/80 mm Hg. What is most likely cause of deterioration?

Explanation

Recall that the complications of ulcers include Bleeding (hemorrhage), penetration, perforation, outlet obstruction and malignization.

Hemorrhage: Hemorrhage is the most common complication of pep­tic ulcer disease. Symptoms include hematemesis (vomiting of fresh blood or \"coffee ground\" material); passage of bloody or black tarry stools (hematochezia and melena, respectively); and weakness, orthostasis, syncope, thirst, and sweating caused by blood loss.

Penetration: A peptic ulcer may penetrate the wall of the stomach or duodenum and enter the adja­cent confined space (lesser sac) or organ (eg, pancreas, liver). Pain may be intense, persistent, referred to sites other than the abdomen (usually the back when caused by penetration of a posterior duode­nal ulcer into the pancreas), and modified by body position.

perforation: perforation usu­ally presents as an acute abdomen. Ulcers that perforate the peritoneal cavity are usu­ally located in the anterior wall of the duo­denum or, less commonly, in the stomach. The patient experiences sudden, intense, steady epigastric pain that spreads rapidly throughout the abdomen, often becoming prominent in the right lower quadrant and at times referred to one or both shoulders.

Gastric outlet obstruction: This may be caused by scarring, spasm, or inflammation associated with an ulcer. Symptoms in­clude recurrent large volume vomiting, oc­curring more frequently at the end of the day and often as late as 6 h after the last meal. Persistent bloating or fullness after eating and loss of appetite also suggest gas­tric outlet obstruction. Prolonged vomiting may cause weight loss, dehydration, and alkalosis.

Malignization/ Stomach cancer: H. pylori is associated with intestinal-type ad­enocarcinoma of the gastric body and an­trum but not cancer of the gastric cardia. Gastric lymphomas and mucosa-associated lym­phoid tissue (MALT) lymphomas have also been linked to this infection.

63. A 57-year-old male patient complains of dyspnea on exertion, heaviness in the right hypochondrium and shin edemata towards evening. Objectively: temperature - 38, 1oC, HR- 20/min, HR=Ps=92/min, AP- 140/90 mm Hg. There is apparent kyphoscoliosis. In the lungs single dry rales can be auscultated. Heart sounds are muffled, rhythmic. ECG: Rv1+Sv5=15 mm. X-ray picture shows the bulging of pulmonary artery cone, right ventricle enlargement. What is the most likely cause of this condition?

Explanation

64. A 40-year-old woman with a history of combined mitral valve disease with predominant stenosis complains of dyspnea, asthma attacks at night, heart problems. At present, she is unable to do easy housework. What is the optimal tactics of the patient treatment?

Explanation

 

A commissurotomy is a surgical procedure used in people with narrowing/ stenosis of heart valves. It involves the removal of deposits on the valve leaflets such as calcium,scar etc.  An artificial valve replacement will be carried in situations of degenerative valve diseases, very low ejection fraction < 30%.

65. A 25-year-old patient complains of having dull heart pain for the last 10 days, dyspnea on mild exertion, palpitations. The diasease developed 2 weeks ago after a respiratory infection. Objectively: acrocyanosis, AP- 90/75 mm Hg, Ps-96/min. Cardiac borders appear to be shifted to the left and right. Heart sounds are weak and have triple rhythm, there is systolic murmur at the apex. ECG showed sinus rhythm, complete left bundle branch block. What is the most likely diagnosis?

Explanation

66. A child undergoes in patient treatment for acute staphylococcal destruction of the right lung. Unexpectedly he developed acute chest pain on the right, dyspnea, cyanosis. The right side of chest lags behind in the respiratory act. Percussion reveals dullness in the lower parts on the right, bandbox resonance in the upper parts. Borders of the relative cardiac dullness are shifted to the left. What complication has most likely developed?

Explanation

67. A 28-year-old patient consulted a surgeon about pain, edema and hyperemia of the left side of his face, weakness, fever up to 39oC. These manifestations has been present for three days. Objectively: there is an inflammatory infiltrate of the left nasolabial fold 4x4 cm large with a necrotic core in the center, the pronounced edema of the left side of face, moderate nuchal rigidity. What treatment is needed?

Explanation

68. A 6-year-old child complains of frequent liquid stool and vomiting. On the 2nd day of desease the child presented with inertness, temperature rise up to 38, 2oC, Ps- 150 bpm, scaphoid abdomen, palpatory painful sigmoid colon, defecation 10 times a day with liquid, scarce stool with mucus and streaks of green. What is a provisional diagnosis?

Explanation

69. A 48-year-old patient complains of having dull pain in the right lumbar region for over three years. USI shows that kidneys are of normal size, at the upper pole of the right kidney there is a fluid containing formation up to 12 cm in diameter. Excretory urograms show normal condition on the left, and the deformation of the superior renal calyces with satisfactory function on the right. What kind of disease can you think of?

Explanation

Recall that a cyst is a fluid filled cavity located in the body. Observe that from the ultrasound investigation, at the upper pole of the right kidney there is a fluid containing formation up to 12 cm diameter - this indicates the presence of a cyst in the right upper pole of the right kidney. Multiple cyst is ruled out because only a single formation was found on the ultrasound investigation. 

 

In hydronephrosis, there will be a build up of urine in the kidneys due to an obstruction/blockage in the bladder.

70. While staying in a stuffy room a 19-year-old emotionally labile girl developed severe weakness, dizziness, blackout, nausea and loss of consciousness without convulsions. Objectively: the patient is unconscious, the skin is pale, extremities are cold. AP- 90/60 mm Hg, Ps- 96/min, deficient, breathing is shallow. Pupillary and tendon reflexes are present. There are no pathological signs. What is the most likely diagnosis?

Explanation

 Notice that the question states ‘loss of consciousness without convulsions’ ‘ this rules out the option of an epileptic attract which is characterised by seizures that sometimes lead to  loss of consciousness. Syncope is simply termed fainting; it occurs as a result of reduced blood flow to the brain; notice that the blood pressure is very low- this leads to a reduced blood flow to major organs of the body especially the brain. This is a leading mechanism for the development of syncope. 

 

Hysterical neurosis is a neurologic disorder characterised by sensory and motor disturbances as well as emotional outbursts and breakdown.

71. A 13-year-old boy complains of pain in the upper third of his left thigh, body temperature rise up to 39oC. There is a swelling in the upper third of thigh and inguinal fold smoothness. The extremity is in a half-bent position. Active and passive movements are not possible because of the sharp pain. What is the most likely diagnosis?

Explanation

Acute hematogenous osteomyelitis is an infection of the bone usually caused by a bacteria and common in children less than 5 years; typically affects the metaphysis of long tubular bones. Children present with fever, localized pain, swelling, and rarely erythema around a long bone, limited range of motion, and limping or refusal to bear weight or use an extremity (pseudoparalysis).

 

Brodie's abscess is a sub-acute form of osteomyelitis, presenting as a collection of pus in bone.  Classically, this may present after progression to a draining abscess extending from the tibia out through the skin.

72. A 12-year-old cild has been hit on the stomach. The patient is in moderately grave condition, has a forced position in bed. The skin is pale, Ps- 122/min. The stress on the left costal arch causes a slight pain. There are positive Weinert, kulenkampff symptoms. Macroscopically the urine is unchanged. What is the most likely diagnosis?

Explanation

 

A positive Kulenkampff’s symptom refers to an acute pain during palpation of the stomach and an absent tension of muscles of the anterior abdominal wall. This sign helps in the diagnosis of abdominal bleeding. The main symptom of a ruptured spleen is pain located around the left costal arch. A kidney related pathology will present with pain in the lumbar region (since the kidney is a retroperitoneal organ) and also have an effect on urine. Pancreatic pathologies will present with belt-like pain. In peritonitis, the pain is dull and poorly localised.

73. A 30-year-old multigravida has been in labour for 18 hours. 2 hours ago the pushing stage began. Fetal heart rate is clear, rhythmic, 136/min. Vaginal examination reveals the completecervical dilatation, the fetal head in the pelvic outlet plane. Sagittal suture in line with obstetric conjugate, the occipital fontanel is near the pubis. The patient has been diagnosed with primary uterine inertia. What is the further tactics of labour management?

Explanation

UTERINE INERTIA  (“Failure to progress”, hypotonic uterine dysfunction) describes lack of progressive cervical dilatation and/or descent of the fetus.  It is such a condition in which uterine contractions strength, duration and frequency are inadequate, that’s why cervical effacement, dilation and fetal descending is slower than in normal labor. Since the baby’s head is already in the pelvic outlet plane and cant proceed further due to inadequate contractions, an outlet forceps is used in assisting delivery.

 

Cesarean session in cases of breech or fetal distress.

74. A patient complains of frequent, bulky, frothy stools with greenish mucus, cramping pain in the umbilical region, abdominal murmur, body temperature at the rate of 39oC. The patient associates the disease with consumption of soft-boiled eggs. What is the most likely pathogen?

Explanation

75. X-ray picture of chest shows a density and an abrupt decrease in the upper lobe of the right lung. The middle and lower lobe of the right lung exhibit significant pneumatization. The right pulmonary hilum comes up to the dense lobe. In the upper and middle parts of the left pulmonary field there are multiple focal shadows. In the basal region of the left pulmonary field there are clear outlines of two annular shadows with quite thick and irregular walls. What disease is this X-ray pattern typical for?

Explanation

76. A patient complains of impaired far vision. Previously his eyes often turned red and hurt. Objectively: the eyes are not irritated, the cornea is transparent, anterior chambers are median deep, their liquid is transparent. The iris of the right eye has not changed in colour, its pattern is unchanged. The pupil is of irregular shape, scalloped. Biomicroscopy of the crystalline lens reveals the areas of opacity and vacuoles. Make a diagnosis:

Explanation

Cataract is the opacification ( clouding ) of the lens which leads to a decrease in vision ( visual acuity). The visual acuity in adults is 60/60 or 6/6 which equals 1. Notice that on examination, the cornea, anterior chamber , iris showed no pathology but there are areas with opacities and vacuoles on the crystalline lens that indicates the presence of cataract. Also notice the irregularities in the pupil ( it is pathologic). A complicated cataract is one accompanied by or caused by an intraocular pathology - in this case the patient’s cataract is accompanied by the defect in the pupil.

Note that senile cataract is age-related ( progressive clouding and thickening of the lens as one gets older).

Tetanic cataract is mostly seen in individuals with hypoglycemia especially after a thyroidectomy. Also referred to as hypocalcemic cataract.

77. During her first visit to the prenatal clinic a pregnant woman was referred to other doctors for mandatory consultation. The patient was refered to:

Explanation

78. A 59-year-old male complains of heart pain, cough, fever up to 38oC. Three weeks ago he suffered a heart attack. Objectively: Ps- 86/min, rhythmic, blood pressure - 110/70 mm Hg. Auscultation reveals pericardial rub, rales beneath the shoulder blade. Radiography reveals no pathology. Blood count: WBCs - 10 · 109/l,ESR - 35 mm/h. ECG shows no dynamics. It would be most reasonable to administer the drugs of the following pharmaceutical group:

Explanation

From anamnesis, this patient presents with fever, pain, a sign of pericarditis- inflammation of the heart pericardium ( the presence of pericardial rub on auscultation) etc. From the listed options of medications to be given, fibrinolytics and coagulants should be given in the case of a clot formation, Nitrates and nitrites are vasodilators; they may relieve the pain but have no effect on fever and the inflammatory process. Antibiotics would have been highly considered but notice that from blood analysis, the wbc is just a little bit above normal while the ESR is within normal range. Recall that glucocorticoids have anti-inflammatory, immunosuppressive and antipyretic effects and in this case will be best suited amongst the available options.
79. A 40-year-old female patient complains of headache, dizziness, muscle weakness, occasional cramps in the extremities. She has been taking anti hypertensive medications for 10 years. AP- 180/100 mm Hg. Blood potassium -1,8 millimole/l, sodium - 4,8 millimole/l. In urine: alkaline reaction, the relative density - 1012, protein and sugar are not found, WBCs - 3-4 in the field of vision, RBCs - 1-2 in the field of vision. Conn’s syndrome is suspected. Which drug should be chosen for the treatment of arterial hypertension?

Explanation

 

While considering the medication for hypertension, we should put in consideration that this patient is experiencing hypokalemia ( low level of potassium in the blood)(Potassium, plasma 3.5-5.0 mmol/L is the normal range) ; note that the alkaline urine reaction is as a result of an increased excretion of potassium in the urine. So while administering the antihypertensive medication, we should also try to reduce or stop the further excretion of potassium. In this case, the best possible option is spironolactone which is a Potassium sparing diuretic ( causes urine excretion but prevents potassium removal). Propranolol ( beta blocker) and enalapril ( sodium channel blocker) are anti hypertensive drugs too but they don't prevent the excretion of potassium. Hydrochlorothiazide is a thiazide diuretic, it reduces sodium reabsorption in the distal convoluted tubule and keeps calcium in the blood.

80. A 27-year-old patient complains of nasal haemorrhages, multiple bruises on the anterior surface of the trunk and extremities, sudden weakness. In blood: Hb- 74 g/l, reticulocytes - 16%, RBCs -2, 5 · 1012/l, platelets - 30 · 109/l, ESR- 25mm/h. What is the most effective measure for the treatment of thrombocytopenia?

Explanation

 

This patient is experiencing multiple hemorrhages; notice that the platelets are very low ( Platelet count 150 − 400 · 109/L- norm) and due to the continuous hemorrhage, the rbc, hemoglobin level and wbc levels are low - this signals the increased production of reticulocytes ( which are immature red blood cells). Recall that the spleen acts as a blood filter, removing damaged or abnormal blood cells. In patients with Immune thrombocytopenia, their immune system treats the blood platelets as foreing bodies, thereby signaling their removal by the spleen leading to thrombocytopenia. Therefore, if the spleen is removed, more platelets can be preserved.

81. After lifting a load a patient felt undurable pain in the loin. He was diagnosed with acute lumbosacral radiculitis. Which of the following is contraindicated for this patient?

Explanation

82. The institutions which take part in medical examinations can be prevention and treatment facilities, medical board of Ministry of Defense, medical board of Ministry of Home Affairs, medico-social expert commissions, forensic medical boards etc. What institutions are responsible for temporary disability examination?

Explanation

83. While lifting a heavy load a 39-year-old patient suddenly felt a severe headache, pain in the interscapular region, and started vomiting. Objectively: the pulse is rhythmic, 60/min, AP- 180/100 mm Hg. The patient is agitated. He presents with photophobia, hyperacusis. There are positive Kernig’s and Brudzinski’s signs on both sides. In blood: WBCs - 10 · 109/l. CSF is bloody, cytosis is 240/3. What is the most likely diagnosis?

Explanation

A positive kernig and brudzinski’s sign indicates the pathology has a  neurological connection. The main finding here is the bloody csf found. This shows that there is bleeding located around either the brain or spinal cord. From the listed options, the only pathology that involves bleeding is subarachnoid hemorrhage. The term subarachnoid hemorrhage (SAH) refers to extravasation of blood into the subarachnoid space i.e. the space between the pia and arachnoid membranes. The subarachnoid space is the space where the cerebrospinal fluid (CSF) circulates, therefore, hemorrhage into this space will result in bloody CSF as mentioned in the question or CSF liquor with high content of erythrocytes. The CSF is responsible for protecting your brain from injury by serving as a cushion. A hemorrhage in this space can cause a coma, paralysis, and even death. It is often characterized by a severe headache in adults (often referred to as ‘worst headache of my life’).  
84. A 45-year-old patient, a sailor, was hospitalized on the 2nd day of the disease. A week ago he returned from India. Complains of body temperature of 41oC, severe headache, dyspnea, cough with frothy rusty sputum. Objectively: the patient is pale, mucous membranes are cyanotic, breathing rate is 24/min, tachycardia is present. In lungs: diminished breath sounds, moist rales over both lungs, crepitation. What is the most likely diagnosis?

Explanation

85. An 8-month-old baby has decreased appetite, pale skin, enlarged right side of abdomen. Palpation the right side of abdomen reveals a dense elastic tumour like formation 10x7 cm large. There is a positive ballotement sign. What is the most likely diagnosis?

Explanation

Apart from orthopedics where the ballotment sign indicates increased fluid in the suprapatellar pouch at the knee joint, the term ‘ballotment’ is also used in referring to an ascitic abdomen. Considering the age of the child, and the result from the palpation of the right side of the abdomen, we can narrow our option to wilms tumor ( nephroblastoma) which is the most common type of kidney tumour ( adrenal medulla) that affects children less than 4 years of age. The tumours in the area of the kidney affects its function leading to reduced fluid excretion via urine resulting in ascites ( accumulation of fluid in the abdomen).
86. An infant is 3 weeks old. Since birth there has been observed periodical vomiting within a few minutes after feeding. The amount of vomitive masses does not exceed that of previous feeding. The infant has age-appropriate body weight. What is the most likely cause of this symptom?

Explanation

 

Pylorospasm is a condition in which the Pyloric sphincter fails to relax properly due to continuous contractions (spasms). The key finding here is that vomiting occurs a few minutes after eating and the volume does not exceed previous food content; this helps differentiate it from Pyloric stenosis which occurs due to the narrowing of the pyloric part of the stomach leading to improper digestion. Pyloric stenosis is characterized by splashing sound in the epigastrium, Projectile vomiting followed by an eagerness to take food. In esophageal achalasia, the patient will firstly experience Dysphagia ( difficulty in swallowing) followed by vomiting; this occurs due to failure in the relaxation of the lower esophageal sphincter and peristalsis.

87. A city somatic hospital with 300 beds consists of the main building which houses the therapeutic and surgical departments. Several separate buildings house the maternity, pediatric and radiologic departments that are connected to the main building by underground walkways and above-ground covered skybridges. Specify the building system of the hospital:

Explanation

88. A 13-year-old girl complains of fever up to 37, 4oC during the last 2 months after recovering from ARVI. Objectively: malnutrition, diffuse grade II enlargement of the thyroid gland feeling dense on palpation, exophthalmos, tachycardia. What kind of pathological syndrome is it?

Explanation

 

From observation, this patient presents with an enlarged thyroid gland, exophthalmos ( eyeballs bulging out), tachycardia. These are characteristic findings in a patient with hyperthyroidism ( thyrotoxicosis- excess circulating thyroid hormone). Tremors, weight loss and fatigue are other vital signs that can be seen in such patients. Hypothyroidism will present with the exact opposite symptoms ie., weight gain, slow metabolic rate, bradycardia etc. The parathyroid gland produces parathormone; a hormone responsible for regulating blood calcium and phosphorus level.

89. A 53-year-old female patient complains of cardiac pain and rhythm intermissions. She has experienced these presentations since childhood. The patient’s father had a history of cardiac arrhythmias. Objectively: the patient is in grave condition, Ps- 220 bpm, AP- 80/60 mm Hg. ECG results: heart rate - 215/min, extension and deformation of QRS complex accompanied by atrioventricular dissociation; positive P wave. Some time later heart rate reduced down to 45/min, there was a complete dissociation of P wave and QRST complex. Which of the following will be the most effective treatment?

Explanation

 

Recall that on the ECG, the P wave represents atrial depolarization ( contraction), QRS complex- ventricular depolarization ( contraction) and T wave - ventricular repolarization ( relaxation). The sinus atrial node is the natural pacemaker of the heart; it produces electrical impulses at the rate of 60-70 beats per minute. Realise that in this patient, at certain periods, she experiences fibrillations while at other times, she experiences bradycardia. Coupled with results from the ECG, we can conclude that this patient has a defective pacemaker. An artificial pacemaker will be the most effective treatment for this patient. Calcium antagonists ( blockers) will only be useful in periods of tachyarrhythmia but are contraindicated in bradycardia.

90. On the 2nd day of life a full term boy developed mild jaundice of skin and mucous membranes, the general condition of the child is normal. Blood test results: indirect hyperbilirubinemia - 120 mmol/l. The child’s blood group is A(II) Rh(+), his mother’s blood group - B(III) Rh(+). What is the doctor’s tactics of choice?

Explanation

 

Notice that the Rh group of both mother and child are the same. This Proves that the increased unconjugated bilirubin is NOT due to Rh incompatibility. This baby is having PHYSIOLOGICAL JAUNDICE; a condition characterised by a high level of unconjugated bilirubin within the first week of life. Mild infant jaundice often disappears on its own within two or three weeks. Moderate and severe  cases are often treated with phototherapy, intravenous immunoglobulin etc. In this case, mild jaundice is experienced therefore, no  drug therapy is needed.

91. A 49-year-old patient complains of dyspnea, cough. There are no sputum discharges. He has repeatedly used salbutamol and intal but with no effect. Objectively: he is only able to sit while leaning on the table. Cyanosis of face, acrocyanosis are present. Breathing is shallow, laboured, in some parts it cannot be auscultated; there are diffuse rales, expiration is significantly prolonged. Heart sounds are muffled, tachycardia is present. Ps - 112/min, AP- 110/70 mm Hg. Liver is located near the costal arch. There are no peripheral edemata. What is your provisional diagnosis?

Explanation

This patient presents with difficulty in breathing, cough, no sputum production, and is cyanotic - these are signs that indicate an asthmatic attack. But also notice that salbutamol has no effect on his current situation ( recall that salbutamol is beta-2 adrenergic receptor agonist usually in the form of inhalers; they cause relaxation of airway smooth muscles). The current state of the patient coupled with unresponsiveness to the medication indicates a state known as Status Asthmaticus. It is defined as a state of severe asthmatic attack which does not respond to conventional treatment. 

 

N/B the fact that this patient does not respond to salbutamol rules out the option of bronchial asthma ( moderate gravity) and cardiac asthma.

92. Blood typing resulted in positive isohemagglutination reaction with standard sera of А(II) and В(III) groups and negative reaction with sera of 0(I) and АВ(IV) groups. What is this result indicative of?

Explanation

Recall that different blood groups present with different antigens and antibodies.

Group I (O) presents with anti A and anti B antibodies but no antigen.

Group II (A) presents with anti B antibody and A-antigen.

Group III (B) presents with anti A antibody and B- antigen

Group IV (AV) presents with no antibodies and antigen A and antigen B. Note that the presented antigen determines the blood group.

 In terms of compatibility, Group IV (AB) is a universal recipient while Group I (O) is a universal donor; Group II (A) can receive from A and O but can donate to A and AB; Group III(B) can receive from B and O but can donate to B and AB.

From the question stem, a positive reaction with Group II (A) and III(B) shows that anti A and B antibodies are present in the sample blood (indicating group I (O)) - but also notice that when the blood sample was tested with group I (O) and IV (AB), a negative reaction occurs. This proves that the standard sera used in testing is faulty and is currently giving inaccurate results because there should be a positive reaction with Blood Group AB (IV) - it  has both A and B antigen.

93. A patient complains of jerking, throbbing pain in the III finger on the right hand. The patient associates these pain onsets with an injury by a nail. The finger skin is hyperemic and tense, palpation with a bulbous-end probe reveals the most painful area. What is the provisional diagnosis?

Explanation

 

A felon ( a form of whitlow) is an acute and painful inflammation of the deeper tissues of the fingers or toes. It is characterised by the presence of an abscess in the affected area. Paronychia is a bacterial or fungal infection of the nail ( where the nail meets the skin). Pandactilitis is an inflammation of the entire finger or toe joints, in this case, the inflamed digits look like sausages.

94. Sanitary examination of the burns unit for adults revealed that 4-bed wards had an area of 28 m2. What is the minimally required ward area for this department?

Explanation

95. A patient presented to a hospital with a carbuncle of the upper lip. The body temperature is 39oC. There is a pronounced edema of the upper lip and eyelids. What is the surgeon’s tactics of choice?

Explanation

 

A carbuncle is a collection of boils ( pus filled inflammation) all connected under the skin. This patient presents to the surgeon with carbuncle and edema of the upper lip and eyelids.  The surgeon should admit this patient, drain the carbuncle, followed by an antibiotic regime with observation of the patient to see if any complication arises.

96. From urine of a 14-year-old boy with the exacerbation of secondary obstructive pyelonephritis Pseudomonas aeruginosa was isolated with a titer of 1000000 microbes per 1 ml. Which antibiotic is most advisable to be administered in this case?

Explanation

 

Pseudomonas aeruginosa , an encapsulated, Gram-negative, rod-shaped bacterium is resistant to a wide range of antibiotics; from the listed drugs, Ciprofloxacin ( a quinolone) is the best choice, It inhibits DNA replication by inhibiting bacterial DNA topoisomerase and DNA-gyrase. Other possible drugs include gentamicin, carbenicillin, carbapenems etc.

97. A 62-year-old male patient complains of intense pain in the left leg that suddenly arose three hours before, leg numbness and coldness. During the year there has been pain in the leg while walking, hypersensitivity to cooling. Objectively: the left foot and shin have marbled skin, subcutaneous veins are collapsed. The foot is cold, active movements of the foot and toes are preserved. Pulse is present only on the femoral artery. There is rough systolic murmur above the artery. Make a provisional diagnosis:

Explanation

 

Duroziez sign is elicited as an audible systolic murmur heard over the femoral artery when the artery is compressed proximally along with a diastolic murmur when the femoral artery is compressed distally. This patient presents with signs of an occluded left femoral artery, these include; pain while walking, numbness and coldness. Also notice that since this issue began just 3 hours before- it is an acute case. The state of the skin of the left foot and shin; a marbled skin is seen when blood supply is not adequate to that area - in this case due to an occlusion to the left femoral artery.

98. A 28-year-old patient complains of profuse, painful and prolonged menstruation. Before and after the menstrual period there is spotting lasting for 4-6 days. Vaginal examination reveals that the uterus is enlarged corresponding to 5-6 weeks of pregnancy, has limited mobility, is painful. Appendages are not palpable. On the 15th day of the menstrual cycle, the uterus was of normal size, painless. On account of stated problems and objective examination the patient has been diagnosed with internal endometriosis. Which drug should be used for the effective treatment of this patient?

Explanation

Endometriosis is a condition in which the tissues lining the endometrium grow outside the uterus in places such as the fallopian tubes, ovaries, etc. It is usually accompanied by intense pain, dysmenorrhea ( painful periods), painful intercourse, excessive bleeding - most of which are seen in the patient described above. Groups of drugs used in treatment of this pathology include: Hormonal contraceptives, Gonadotropin-releasing hormone (Gn-RH) agonists and antagonists, progestin or combination hormonal contraceptive. 

Duphaston (dydrogesterone) is a progestin ( progesterone)  indicated in endometriosis, infertility, premenstrual syndrome etc 

Synoestrolum is a non steroidal estrogen indicated in breast and prostate cancer.

 

Parlodel (Bromocriptine) is used in hyperprolactinemia.

99. A 24-year-old female patient complains of pain in the right hypochondrium that is getting worse after taking meals; nausea, fever up to 37, 7oC, icteric skin, pain in the large joints. These presentations have been observed for 8 months. Objectively: hepatosplenomegaly. Blood test results: ESR- 47 mm/h, total bilirubin - 86,1 mmol/l, direct bilirubin - 42,3mmol/l. Total protein - 62 g/l, albumins- 40%, globulins - 60%, gamma globulins - 38%. Viral hepatitis markers were not detected. The antibodies to smooth muscle cells are present. On ultrasound the portal vein diameter was of 1 cm. What is the most likely diagnosis?

Explanation

 

Presence of pain in the right hypochondrium should indicate that the problem lies either in the liver, gallbladder or bile ducts. Coupled with an increased level of bilirubin, antibodies against smooth muscles were found from biochemical analysis. Note that Smooth muscles antibodies are  found in autoimmune type hepatitis. This is because certain liver diseases such as hepatitis and cirrhosis trigger the body to form antibodies against smooth muscle. Other antibodies that can be seen in autoimmune hepatitis include Antinuclear antibodies, Anti-actin antibodies, Anti-soluble liver antigen/liver pancreas (anti-SLA/LP) antibodies. Other antibodies may be present, even when the anti-smooth muscle antibodies are absent.

100. A patient is 60 years old, retired, worked as deputy director of a research institute. Behavioural changes appeared 2 years ago after the death of her husband: she stopped looking after herself and leaving the house; then she refused to clean the apartment and cook. Mental status: temporal disorientation. The patient does not understand many of the questions, is confused; does not know how to cook soup or fasten a button. Her speech is characterized by stumbling and logoclonia. She does not recognize doctors, fellow patients. She cries a lot but can not explain the reason for tears. What is the mechanism of this pathology?

Explanation

 

Recall that the cerebral cortex is made up of different lobes, sulci and gyri that act as centres for various functions for example the occipital lobe for vision. The cerebral cortex can further be divided into brodmann's area with each area having a specific function eg area 44 & 45 ( broca’s area) - for motor activities in speech production etc. this patient presents with temporal disorientation, problems with understanding questions, repetition of syllables (logoclonia), long term memory defects- these signs are mostly due to a defect in the Temporal lobe ( responsible for long term memory, processing auditory and visual sensory input, language recognition and formation of new memories). Note that from anamnesis, since this patient does not have any underlying disease that can lead to an atherosclerotic formation, that option is ruled out. The above stated symptoms are most likely due to the atrophy of the cerebral cortex.

101. A 70-year-old patient with a strangulated inguinal hernia called a therapist in. The strangulation took place 10 hours ago. There are signs of intestinal obstruction. The skin over the herniation is hyperemic. What is the tactics of choice?

Explanation

 

A strangulated hernia is a life threatening condition. This is because this form of hernia cuts off blood supply to the intestines and tissues of the abdomen; it can be complicated by perforation of the intestine, shock, gangrene etc. This condition requires a surgical emergency.

102. An employee was on a business trip to another city, where he fell ill and was hospitalized. The sick leave certificate can be issued:

Explanation

103. A 14-year-old boy with a history of chronic tonsillitis and sinusitis has developed a feeling of heart irregularities and additional pulse. HR- 83/min. ECG results: regular impulses with no visible P wave that occur every two sinus contractions, QRS complex is dramatically deformed and prolonged to over 0,11 s, T wave is discordant followed by a complete compensatory pause. Specify the arrhythmia type:

Explanation

 

Note that from the results from ECG- regular impulses with no visible P wave that occur every two sinus contractions. This indicates a case of Premature ventricular contractions ( PVC); When this occurs in a three-beat pattern, it is called trigeminy. This pattern can be two normal (sinus) beats and one abnormal one. Another trigeminy pattern is two PVCs with one sinus beat. This rhythm is different from bigeminy, where the heart beats with one sinus beat and one PVC. 

104. An 8-year-old girl periodically has sudden short-term heart pain, sensation of chest compression, epigastric pain, dizziness, vomiting. Objectively: the patient is pale, respiratory rate - 40/min, jugular pulse is present. Ps- 185 bpm, of poor volume. AP- 75/40 mm Hg. ECG taken during an attack shows ectopic P waves, QRS wave is not deformed. At the end of an attack a compensatory pause is observed. The most likely cause of the attack is:

Explanation

105. A 45-year-old patient with acute abscess of the left lung has suddenly developed acute chest pain and dyspnea while coughing, tachycardia has increased. The control Ro-gram shows collapsed left lung, the air in the left pleural cavity and a horizontal fluid level. What is the mechanism of this complication?

Explanation

 

From anamnesis, there is an abscess ( fluid filled cavity) in the left lung. The development of sudden pain, dyspnea and tachycardia raises a question about the worsening of the patient’s current state. The result from the Ro-gram ( pulmonary function study) shows the presence of air and fluid in the pleural cavity of the left lung thereby indicating that the earlier mentioned abscess has burst open and its content has spread into the pleural space.

106. Examination of a 38-year-old patient who had been hit with a blunt object on the left side of chest revealed a fracture of the X rib with fragments displacement, parietal pneumothorax. The patient complains of pain in the left subcostal area. Objectively: the patient is pale, AP- 80/40 mm Hg, Ps- 138/min, of poor volume. USI reveals fluid in the left abdomen. Splenic rupture is present. What treatment tactics should be chosen?

Explanation

107. A baby is 3 months old. The mother consulted a pediatrician about lack of breast milk. After several test weighings it was found that the child had to receive supplementary feeding. What is the optimal milk formula for this child?

Explanation

108. Examination of a newborn revealed skin redness that appeared immediately after birth and reached the maximum intensity on the second day of life. What is your provisional diagnosis?

Explanation

Note that this baby is actually experiencing Erythema toxicum neonatorum (ETN); an erythemic process usually seen in full term infants. It starts on the first day after birth and usually fades away within a week. 

Erythema nodosum is a type of skin inflammation that is located in a part of the fatty layer of skin. Erythema nodosum results in reddish, painful, tender lumps most commonly located in the front of the legs below the knees. The tender lumps, or nodules, of erythema nodosum range in size. 

 

Transient erythema occurs as a result of loss of hair ( maybe due to exposure to radiation) and neurological changes.

109. A 27-year-old sexually active female complains of numerous vesicles on the right sex lip, itch and burning. Eruptions regularly turn up before menstruation and disappear 8-10 days later. What is the most likely diagnosis?

Explanation

Herpes simplex viruses are enveloped double stranded linear viruses. HSV-1 is also known as oral herpes ( gingivostomatitis, keratoconjunctivitis etc) while HSV-2 is known as genital or neonatal herpes. The clinical presentation of genital herpes include  pain, itching, dysuria, vaginal and urethral discharge, tender lymphadenopathy, appearance of herpes vesicles on the  external genitalia, labia majora, labia minora, vaginal vestibule - for women and glans penis, the prepuce, the shaft of the penis, and sometimes on the scrotum, thighs, and buttocks- for men.

Primary syphilis usually presents with a localized painless hard chancre on the genitals.

 

Cytomegalovirus is also known as human herpes virus-5, it is usually seen in immunocompromised patients and infected cells have characteristic ‘ owl's eye’ intranuclear inclusions. 

110. A 38-year-old patient has suddenly developed pain in the left side of his chest, suffocation. Objectively: moderately grave condition, Ps- 100/min, AP- 90/60mm Hg, breath sounds on the left cannot be auscultated. Chest radiography shows the collapse of the left lung up to 1/2. What kind of treatment should be administered?

Explanation

This patient is currently having a spontaneous pneumothorax. Spontaneous pneumothorax refers to the sudden collapse of the lungs in the absence of any apparent cause such as trauma or any underlying lung pathology. Notice that the chest radiography shows the collapse of the left lung indicating a spontaneous pneumothorax. This is an emergency procedure that requires a Thoracostomy to be carried out ( a small surgical incision that allows the drainage of air, fluid etc). 

The type of drainage selected depends on the cause and type of pleural disease.

Passive drainage relies on the increased intrathoracic pressure generated during exhalation. This will force air from the pleural space. This has been shown to be very effective in treating spontaneous pneumothorax and traumatic pneumothorax. There is no active suction on the lungs, the theory being this will allow small ruptures in the pulmonary parenchyma to heal whereas constant suction could dislodge the fibrin and lead to delayed healing of the tear in the pulmonary tissue.

 

Active drainage entails suction consistently placed on the thoracostomy tube and works well in the case of marked pleural effusion and pyothorax.

111. A 40 week pregnant secundipara is 28 years old. Contractions are very active. Retraction ring is at the level of navel, the uterus is hypertonic, in form of hourglass. On auscultation the fetal heart sounds are dull, heart rate is 100/min. AP of the parturient woman is 130/80 mm Hg. What is the most likely diagnosis?

Explanation

Hysterorrhexis is the violation of uterus integrity in any part during pregnancy or delivery ( the rupture of the uterus). hysterorrhexis during pregnancy can be explained by inconsistency of the presenting fetal part dimensions with the dimensions of the mother's pelvis. If there is an obstacle to fetus expulsion,rapid birth activity develops, the superior uterine segment contracts more and more, the fetus gradually moves into the thin-walled stretched inferior segment. It's super distension and rupture happen especially easily if the uterine neck has not moved behind the fetal head and is jammed between it and a pelvic wall. At that, the contraction ring reaches the level of the navel; the uterus acquires the form of an hourglass. If birth activity is pro- longed, super distension and thinning of the inferior reaches the highest level, its rupture takes place.

 

In the case where the rupture has already occurred, the clinical presentation is conditioned by uterine wall integrity violation, blood vessel rupture, hematoma/bloody appearance in the myometrium.

112. A 10-year-old patient has a history of mild bronchial asthma. During a regular check-up the patient should be recommended:

Explanation

 Bronchial asthma is a medical condition classified under type 1 hypersensitivity ( IgE mediated); it is a chronic inflammatory pulmonary disease that is characterized by periodic cough, wheezing, tachypnea, dyspnea, hypoxemia, increased production of mucus which blocks the airways. Triggers include stress, allergens, viral upper respiratory infections, cold air etc. On objective examination, Tachypnoe, audible wheezing, hyperinflated chest and hyper resonant sound on percussion are observed. Curshman’s spirals ( shed epithelium in mucus plugs) and Charcot leyden crystals ( broken down eosinophils) are key findings morphologically.
113. A 26-year-old patient with left lower lobe pneumonia experiences an acute chest pain on the left during coughing. Objectively: diffuse cyanosis, extension of the left side of chest. Percussion reveals high tympanitis. Auscultation reveals no respiratory murmurs above the left side of chest. There is a deviation of the right cardiac border towards the midclavicular line. What examination will be the most informative?

Explanation

114. A baby was born by a young smoker. The labour was complicated by uterine inertia, difficult delivery of the baby’s head and shoulders. The baby’s Apgar score was 4. Which of the following is a risk factor for a spinal cord injury?

Explanation

Recall that the spinal cord is connected to the brain via the brainstem. During delivery, injury to the spinal cord/brainstem is likely to occur during expulsion of the head and shoulders of the baby. Apgar score refers to  a rapid method for assessing a neonate immediately after birth and in response to resuscitation. Elements of the Apgar score include color, heart rate, reflexes, muscle tone, and respiration. Each element is scored 0 (zero), 1, or 2. The score is recorded at 1 minute and 5 minutes in all infants with expanded recording at 5-minute intervals for infants who score seven or less at 5 minutes, and in those requiring resuscitation as a method for monitoring response. Scores of 7 to 10 are considered great. 
115. A 10-year-old child has been admitted to a hospital with a closed craniocerebral injury with suspected cerebral edema. The patient is in grave condition, unconscious. The dyspnea, tachycardia, hypertension are present. Muscle tone is increased, there is nystagmus, pupillary and oculomotor reactions are disturbed. The mandatory component of intensive care is dehydration. What diuretic is adequate in this case?

Explanation

By definition cerebral edema is the excess accumulation of water in the intra-and/or extracellular spaces of the brain. The most rapid and effective means of decreasing tissue water and brain bulk is osmotherapy. Osmotic therapy is intended to draw water out of the brain by an osmotic gradient and to decrease blood viscosity. These changes would decrease ICP and increase cerebral blood flow (CBF). 

Mannitol is the most popular osmotic agent. However, prolonged administration of Mannitol results in an electrolyte imbalance that may override its benefits and that must be carefully monitored. Nursing care of the patient receiving Mannitol requires vigilant monitoring of electrolytes and overall fluid balance and observation for the development of cardiopulmonary complications in addition to neurological assessment.

 

Therefore, the best choice is to go for diuretics to prevent some of the complications associated with Mannitol. Furosemide is a loop diuretic with a strong osmotic effect. It has been shown to prolong the reversal of blood brain osmotic gradient established with the osmotic agents by preferentially excreting water over solute. Furosemide is a strong diuretic and acts fast.

116. A 57-year-old patient taken to the surgical department by ambulance has been provisionally diagnosed with acute intestinal obstruction. Acute pancreatitis is suspected. What is the most informative method of study to verify the diagnosis?

Explanation

 

Intestinal obstruction refers to the blockage of food from getting into the small intestine and colon. These blockades can be as a result of strictures ( from an inflammatory condition), medications, fibrotic changes etc. Abdominal cramps, bloating, constipation, inability to pass stool are key findings in this condition. For confirmation of diagnosis of intestinal obstruction, an abdominal X-ray is recommended. Although not all obstructions can be viewed via an abdominal x-ray, additional examinations include abdominal ultrasound and CT.

117. After myocardial infarction, a 50-year-old patient had an attack of asthma. Objectively: bubbling breathing with frequency of 32/min, cough with a lot of pink frothy sputum, acrocyanosis, swelling of the neck veins. Ps- 108/min, AP- 150/100 mm Hg. Heart sounds are muffled. Mixed moist rales can be auscultated above the entire lung surface. What drug would be most effective in this situation?

Explanation

118. A baby born after fast labour has palsy of hand muscles. Grasp reflex is absent, as well as hand-to-mouth reflex.  Hand sensitivity is absent. What is the most likely diagnosis?

Explanation

Dejerine Klumpke’s palsy occurs when there is a traction or tear of the lower trunk: C8-TH1 roots. In infants, it is caused as a result of upward force on the arm during delivery while in adults it occurs after trauma. This condition affects the intrinsic hand muscles ( lumbricals, interossei, thenar and hypothenar). This condition is characterised by a total claw hand ( see image ) because the lumbrical muscles normally flex the metacarpophalangeal joints and extend the Distal and proximal interphalangeal joints.

In Duchenne Erb’s palsy ( waiter’s tip hand), there is a traction or tear to the upper trunk: C5-C6 roots. It is caused by the lateral traction of the neck during the delivery of infants. It affects the deltoid (abduction), Infraspinatus ( lateral rotation) and biceps brachii ( flexion and supination).

Horner’s syndrome is characterised by Ptosis ( slight drooping of the eyelid), Anhidrosis ( absence of sweating) and Miosis ( pupil constriction)

119. Against the background of angina a patient has developed pain in tubular bones. Examination revealed generalized enlargement of lymph nodes, hepatolienal syndrome, sternalgia. In blood: RBCs- 3, 6 · 1012/l, Hb- 87 g/l, thrombocytes- 45 · 109/l, WBCs - 13 · 109/l, blasts -87%, stab neutrophils - 1%, segmented neutrophils - 7%, lymphocytes - 5%, ESR- 55 mm/h. What is the most likely diagnosis?

Explanation

Leukemias are malignant neoplasms of the hematopoietic stem cells characterized by diffuse replacement of the bone marrow by neoplastic cells.

From the question stem, using the full blood count, we can denote that there is Leukemia due to the presence of immature blast cells. To distinguish between Acute and  Chronic leukemia, the blast count is always very important. Blasts are immature white blood cells.

In chronic Leukemia, the blood and bone marrow contain less than 10% blasts (blast - 2%). In Acute leukemia, blasts are usually more than 20%. Using the lab values given in the question, we have 87% of blast cells, which makes it Acute Leukemia..

 
120. In order to reduce weed growth on agricultural land, some herbicides have been used for a long time. In terms of environmental stability these herbicides are rated as stable. Specify the most likely route of their entry into the human body:

Explanation

 

Herbicides are chemicals ( a form of pesticides) used in killing unwanted plants; when applied, they come in contact with the soil and may affect food products (plants) which will cause harm to the consumer. ( soil-plants-humans).

121. A selective population research study was aimed at exploring the effect of air emissions from a metallurgical plant on the obstructive bronchitis morbidity in a city. The calculated correlation coefficient was +0,79. Evaluate the strength and direction of the relationship:

Explanation

The Spearman's Rank Correlation Coefficient is used to discover the strength of a link between two sets of data. This example looks at the strength of the link between the effect of air emissions from a metallurgical plant on obstructive bronchitis morbidity.

Correlation is a bivariate analysis that measures the strength of association between two variables and the direction of the relationship.  In terms of the strength of relationship, the value of the correlation coefficient varies between +1 and -1.  A value of ± 1 indicates a perfect degree of association between the two variables.  As the correlation coefficient value goes towards 0, the relationship between the two variables will be weaker.  A correlation coefficient of zero indicates that no relationship exists between the variables. The direction of the relationship is indicated by the sign of the coefficient; a + sign indicates a positive/direct relationship and a – sign indicates a negative relationship.

An inverse correlation, also known as negative correlation, is a contrary relationship between two variables such that when the value of one variable is high then the value of the other variable is probably low.

The value of the correlation coefficient ‘r’ can range from 0.0, indicating no relationship between the two variables, to positive or negative 1.0, indicating a strong linear relationship between the two variables. 

Value of r

Indications

0.0

No linear relationship between the two variables

+1.0

Strong positive linear relationship; as X increases in value, Y increases in value also; or as X decreases in value, Y decreases also.

-1.0

Strong inverse linear relationship; as X increases in value, Y decreases in value; or as X decreases in value, Y increases in value.

 

Therefore, a value of +0.79 is very close to +1.0 and that will give us a strong positive/direct relationship.

122. A general practitioner visited a 2-year-old child and diagnosed him with measles. The child attends a nursery, has a 5-year-old sister. What document must be filled in for the effective antiepidemic measures in the given health locality?

Explanation

This Patient has a measles infection ( an infectious disease).  A form that will notify the infectious disease department and provide the necessary information to help curtail the spread should be filled
123. Hygienic expertise of a sample taken from the batch of grain revealed that 2% of grains were infected with microscopic Fusarium fungi. On the ground of laboratory analyses this batch of grain should be:

Explanation

124. Examination of a 43-year-old man objectively revealed pallor of skin and mucous membranes, loss of tongue papillae, transverse striation of fingernails, cracks in the mouth corners, tachycardia. Blood test results: Hb- 90 g/l, anisocytosis, poikilocytosis. The most likely causative agent of this state is inadequate intake of:

Explanation

Iron deficiency anaemia is a condition where a lack of iron in the body leads to a reduction in the number of red blood cells. Iron is used to produce red blood cells, which help store and carry oxygen in the blood. If you have fewer red blood cells than is normal, your organs and tissues won't get as much oxygen as they usually would. Causes include blood loss ( bleeding, menorrhagia etc), poor diet, malabsorption and hookworms. Key findings include; koilonychia ( spoon shaped, brittle nails), atrophic glossitis ( loss of filiform papillae on the tongue), angular cheilitis (inflammation of mouth corners) etc. Note that Anisocytosis refers to the presence of RBCs of various sizes while poikilocytosis refers to RBCs of various shapes - both findings are usually in Iron deficiency anemias. 

 

Copper deficiency will present with signs such as difficulty in learning and forming memories, weak and brittle bones, difficulty walking, vision loss (Kayser–Fleischer ring in eyes). 

125. Factory’s sectorial doctor chooses a group of chronically ill people. He takes into account the duration of etiologically related cases with a temporary disability over the last year in each of the workers. The employees will fall into this group if the duration is:

Explanation

126. 6 people live in a modern flat with the total area of 60 m2. There are TV-video equipment, radios, microwave ovens, computer. The residents of the flat complain of bad health, occasional headaches, arrhythmia, conjunctivitis. What is the most likely cause of this condition?

Explanation

 

Many household appliances produce electromagnetic fields for example, television, microwave oven, radios etc. The effects of electromagnetic waves on health are very broad. Immune, neurological, endocrine systems are majorly affected.

127. A man abused alcohol, drank away the property and wages thus getting himself, his wife and two underage children into deep financial problems. He was registered in a local drug abuse clinic. His wife asked a family doctor, what kind of petition she could file in court:

Explanation

128. A child is 12 years old. He complains of a dull aching pain in the epigastrium and right hypochondrium, that is getting worse after taking fatty or fried food, headache, weakness, nausea, low grade fever. Abdominal palpation reveals a marked resistance of muscles in the right hypochondrium, positive Kerr’s, Ortner’s, Murphy’s symptoms. What is the most likely diagnosis?

Explanation

Kehr’s sign- pain in the left shoulder ( due to irritation of the under surface of the diaphragm).

Murphy’s sign- pain during inspiration while palpating the right subcostal region.

Ortner’s sign-  painfulness at the easy pushing on the right costal arch by the edge of the palm. 

These signs are positive in conditions related to the gallbladder eg cholecystitis, cholelithiasis etc.

Chronic cholecystitis refers to a long term inflammation of the gallbladder. It is characterised by flatulence, abdominal distension and discomfort, nausea and fat intolerance. Diagnosed with the help of ultrasound. 

Acute Pancreatitis  will be accompanied by epigastric or central abdominal pain that radiates to the back and is relieved  when sitting forward. Periumbilical bruising ( Cullen’s sign) or on the flanks ( Grey turner’s sign) are key findings in this pathology.
129. A 3-month-old girl presents with rhinitis, dyspnea, dry cough. These manifestations has been observed for two days. Objectively: the child has pale skin, acrocyanosis, shallow respiration at the rate of 80/min. Percussion reveals handbox resonance over the whole surface of lungs, massive fine

Explanation

 

Note that acute bronchiolitis and Acute bronchitis are very similar and basically present with almost the same symptoms. One key difference is the age range of people affected ; Bronchitis is found in individuals of all age ranges although mostly seen in older children and adults  while bronchiolitis is only found in younger children especially below 2 years. From analysis, we hear vesicular tympanic sound and bubbling rales / crackles - these can  be found in both diseases; but the key difference here is the age of the child (3 months).

130. A patient had 4 generalized convulsive seizures within a day. Between the seizures the patient did not maintain clear consciousness (was in a coma or stupor). Specify his state:

Explanation

Status epilepticus (SE) is a common, life-threatening neurologic disorder that is essentially an acute, prolonged epileptic crisis. Status epilepticus is defined as a continuous seizure lasting more than 30 min, or two or more seizures without full recovery of consciousness between any of them. Based on recent understanding of pathophysiology, it is now considered that any seizure that lasts more than 5 min probably needs to be treated as SE.

It can also be defined as a seizure that persists for a sufficient length of time or is repeated frequently enough that recovery between attacks does not occur. Therefore, the key phrase here is “recovery between attacks does not occur.” The question puts it this way - ‘between the seizures the patient did not come to waking consciousness.’

Prolonged Status epilepticus can lead to cardiac dysrhythmia, metabolic derangements, autonomic dysfunction, neurogenic pulmonary edema, hyperthermia, rhabdomyolysis, and pulmonary aspiration. Permanent neurologic damage can occur with prolonged SE.
131. A 45-year-old patient complains of fever up to 40oC, general weakness, headache and spasmodic contraction of muscles in the region of a shinwound. The patient got injured five days ago when tilling soil and didn’t seek medical attention. What kind of wound infection can be suspected?

Explanation

132. A 60 year-old female has been suffering weakness, dizziness, fatigue over the last year. Recently she has also developed dyspnea, paresthesia. Objectively: skin and mucous membranes are pale and slightly icteric. The tongue is smooth due to the loss of lingual papillae. Liver and spleen are located at the costal margin. Blood count: Hb- 70 g/l, RBCs - 1, 7·1012/l, colour index - 1,2, macrocytes. Administer the patient a pathogenetically justified drug:

Explanation

From the blood analysis given, Macrocytes are present- this refers to irregular large Red Blood cells. Also the color index is elevated. The erythrocyte level is also very low ( norm- Female: 3.5 − 5.5 · 1012/L) - this indicates an anemia . The major causes of macrocytic anemia include Vitamin B12 deficiency, Vitamin B9 (Folic Acid) deficiency or medications such as antiretroviral drugs. 

Symptoms include loss of appetite, brittle nails , pale skin, fatigue etc. To differentiate between B12 or Folic acid deficiency, Vit B12 deficiency is often associated with nervous system symptoms like paresthesias - subacute combined degeneration due to abnormal myelin synthesis. These nervous system symptoms are absent in Folic Acid Deficiencies.

With the symptoms listed in the question, and the presence of anemia and macrocytes - it safe to say Vitamin B12 deficiency is the best choice.

Ascorbic acid or vitamin C deficiency will lead to scurvy, petechial bleeding or bruises are common in this case. Vitamin B1 deficiency will lead to the development of Beri-Beri. Iron deficiency will result in Anemia but macrocytes will be absent on the blood film, instead we will see microcytes.

 
133. Analysis of organization of medical care in a regional centre has shown that every year about 12% of patients receive inpatient care for diseases that don’t require round-the-clock monitoring and intensive care. What are the most appropriate organizational changes required to address this problem?

Explanation

134. A newborn (mother’s I pregnancy) weighing 3500 g presents with jaundice, lethargy, reduced reflexes. Objectively: second grade jaundice of skin with saffron tint, liver - +2cm, spleen - +1 cm. Urine and feces are yellow. Blood count: Hb- 100g/l, RBCs - 3, 2 · 1012/l, WBCs - 18, 7 · 109/l, mother’s blood type - 0(I) Rh(+), baby’s blood type - А(II) Rh(-), bilirubin - 170mmol/l, indirect fraction. ALT, AST rates are normal. What disease is the child most likely to have?

Explanation

Hemolytic disease of the newborn is a disease characterised by the increased breakdown of Red blood cells (RBC) in the newborn. Hemolytic disease of the newborn occurs when the immune system of the mother sees a baby's RBCs as foreign. Antibodies then develop against the baby's RBCs. These antibodies attack the RBCs in the baby's blood and cause them to break down too early. Two major hemolytic pathologies in pregnancy are: ABO incompatibility and rhesus incompatibility (Mother Rhesus -ve and child Rhesus +ve)

In this case, the fetus and mother present with ABO incompatibility; Notice that the mother’s blood type is 0(I) while the newborn’s A (II). Antibodies are formed against the newborn’s RBCs leading to its increased destruction. 

 

It is not Rhesus incompatibility because the mother is Rhesus positive while the baby is negative. Rhesus incompatibility occurs when a Rhesus negative (Rh-) mother marries a Rh+ Man and they conceive a Rh+ child. Note: the mother must be Rh- and the child Rh+. The first child with Rh+ usually survives, but subsequently Rh+ fetus will be attacked by Rh antibodies in the mother which crosses the placenta to attack the fetal red blood cells causing hemolysis.

135. Doctors of a polyclinic conduct a statistical research of the disease outcomes in two groups of patients (those registered with dispensary departments and unregistered ones), depending on age and level of hygiene. What type of statistical tables would be most suitable for profound analysis of the interrelation between the above-mentioned variables?

Explanation

136. A 57-year-old female complains of having a sensation of esophageal compresion, palpitation, difficult breathing during eating solid food, occasional vomiting with a full mouth, \\\"wet pillow\\\"sign at night for the last 6 months. Objectively: body tempearture - 39oC, height - 168 cm, weight - 72 kg, Ps- 76/min,АP- 120/80 mm Hg. X-ray revealed a considerable dilation of esophagus and its constriction in the cardial part. What pathology is most likely to have caused dysphagia in this patient?

Explanation

Esophageal achalasia is an esophageal motility disorder involving the smooth muscle layer of the esophagus and the lower esophageal sphincter (LES). It is characterised by an increased LES tone, incomplete LES relaxation and lack of esophageal peristalsis. The sign of “wet pillowresults from increased salivation and nocturnal discharge of saliva and mucus from the mouth and is observed in esophageal pathologies. Diagnosis is confirmed by x-ray examination and from the above patient, we see the dilation of esophagus and its constriction in the cardial part which confirms our diagnosis.  

137. A 37-year-old patient complains of acute pain in the region of genitals, swelling of the labia, pain when walking. Objectively: body temperature is 38, 7oC, Ps- 98/min. In the interior of the right labia there is a dense, painful tumour-like formation 5,0x4,5 cm large, the skin and mucous membrane of genitals is hyperemic, there are profuse foul smelling discharges. What is the most likely diagnosis?

Explanation

Bartholin's glands are located at the openings of the vagina (left and right sides). The Bartholin’s glands are on each side of the vaginal opening. They’re about the size of a pea. They make fluid that keeps the vagina moist.

Acute bartholinitis is a disease usually seen in women in the period of genital activity. Its occurrence in a prepubertal child is an extremely rare event. Bartholinitis is an inflammation of Bartholin’s gland (large gland of vaginal vestibule). It may be caused by Staphylococcus, E.coli and N. gonorrhoeae.. There can be serous, serous-purulent, or purulent inflam­mation. It can manifest as acute, painful unilateral labial swelling, Dyspareunia, Pain with walking and sitting. 

 

Bartholinitis and bartholin cyst are used interchangeably but most cysts are caused by the blockade of the ductal path of the gland leading to an accumulation of its contents. Patients with cysts may present with painless labial swelling.

138. A 28-year-old female patient has been admitted to a hospital. She states to be ill for 12 years. On examination she has been diagnosed with bronchiectasis with affection of the left lower lobe of lung. What is the optimal treatment tactics for this patient?

Explanation

Bronchiectasis is the abnormal and persistent dilatation of the smaller bronchi and mainly the segmental or subsegmental bronchi due to destruction of elastic tissues and muscles of the bronchial wall.

Social limitations in addition to treatment failure, despite preventive precautions and aggressive medical treatment, are the main indications for surgery in patients with bronchiectasis. For a patient that has lived with this condition for 12 years, we can safely assume that she must have tried conservative management which definitely did not work as we can still see exacerbations of the symptoms.

Cases with long-term treatment lead a life without comfort and with high risk due to life-threatening complications. It is important to perform surgical treatment when indicated to save patients from these risks and to minimize the surgical risks in delayed cases.

 

The left lower lobe is affected so the surgery will be a resection of the left lower lobe - left lower lobectomy.

139. 4 weeks after myocardial infarction a 56-year-old patient developed acute heart pain, pronounced dyspnea. Objectively: the patient’s condition is extremely grave, there is marked cyanosis of face, swelling and throbbing of neck veins, peripheral pulse is absent, the carotid artery pulse is rhythmic, 130 bpm, AP is 60/20 mm Hg. Auscultation of heart reveals extremely muffled sounds, percussion reveals heart border extension in both directions. What is the optimal treatment tactis for this patient?

Explanation

Ventricular rupture is a known complication of Myocardial infarction and this patient currently presents with signs and symptoms of cardiac tamponade - extravasation of blood into the pericardial cavity preventing the heart from pumping properly.

 

We can pick out: muffled heart sounds; low blood pressure (hypotension 60/20mmHg); elevated jugular venous pressure (evidenced by swelling and throbbing of the neck veins). These three signs are known as Beck’s triad. They indicate a Cardiac Tamponade (the impaired pumping ability of the heart due to accumulation of fluid in the pericardium). Note that percussion reveals heart border extension in both directions. The most suitable approach  should be a pericardial puncture (pericardiocentesis) to remove the fluid in the pericardium. A thoracotomy can be carried out to gain access to the pleural cavity.

140. A 15-year-old boy feels pain in the region of the left knee joint. Objectively: the soft tissues in the affected region are infiltrated, the joint function is limited. Radiography reveals a focus of bone destruction in the distal metaepiphysial segment of the left femur. The destruction is accompanied by periosteal detachment and a defect formed within cortex of Codman triangle bone. X-ray of chest shows multiple microfocal metastases. What is the most likely pathology?

Explanation

 

Osteosarcoma is a metaphyseal bone tumor. It is the most common type of cancer that arises in bones, and it is usually found at the end of long bones, often around the knee. Presents as a painful enlarging mass or with pathologic fractures. A codman Triangle (elevation of the periosteum)  or sunburst pattern are pathognomonic and classical findings on radiography.

141. On the first day after a surgery for diffuse toxic goiter a patient developed difficulty breathing, cold sweats, weakness. Objectively: pale skin, body temperature - 38, 5oC, RR - 25/min, Ps- 110/min, AP- 90/60 mm Hg. What early postoperative complication occurred in the patient?

Explanation

A patient that underwent surgery for diffuse toxic goiter presented with a high metabolic rate (elevated body temperature, tachycardia, tachypnea) 24hrs post-operation. This patient currently has a thyrotoxic crisis.

One of the rarer complications from thyroid surgery is precipitation of a thyroid storm, which can occur intraoperatively or postoperatively. It is thought to occur secondary to thyroid gland manipulation in the operating room in patients with hyperthyroidism. 

Thyroid (or thyrotoxic) storm is an acute, life-threatening syndrome due to an exacerbation of thyrotoxicosis. Classic features of thyroid storm include fever, marked tachycardia, heart failure, tremor, nausea and vomiting, diarrhea, dehydration, restlessness, extreme agitation, delirium or coma.

Hypothyroid crisis will present with bradycardia, low body temperature, low respiratory rate, constipation etc. Postoperative tetany is a possible postoperative complication in thyroid surgery as the parathyroid gland which resides behind the thyroid could be accidentally resected. Postoperative tetany indicates low calcium levels of which it will most likely be associated with bradycardia instead of the 110bpm (tachycardia) seen in this patient.  

 
142. A 49-year-old patient consulted a doctor about difficult swallowing, voice hoarseness, weight loss. These symptoms have been gradually progressing for the last 3 months. Objectively: the patient is exhausted, there are enlarged supraclavicular lymph nodes. Esophagoscopy revealed no oesophageal pathology. Which of the following studies is most appropriate in this case?

Explanation

 

A CT scan of the chest can help find problems such as infection, lung cancer, blocked blood flow in the lung (pulmonary embolism), and other lung problems. It also can be used to see if cancer has spread into the chest from another area of the body. Note that this patient most likely has cancer since there has been loss of weight over 3 months, progressing symptoms coupled with enlarged lymph nodes around this area. A lung x-ray gives a pictorial view of the heart, lungs, airways, blood vessels and the bones of the spine and chest but a CT scan is more detailed. Because it is able to detect very small nodules in the lung, chest CT is especially effective for diagnosing lung cancer at its earliest, most curable stage. Imaging of the esophagus is not needed as the question clearly stated that the esophagus revealed no pathology.

143. A 45-year-old female patient complaining of general weakness, nausea and vomiting has been delivered to ah hospital by the ambulance. Recently there has been a lack of appetite, weight loss. Objectively: hyperpigmentation of skin, blood pressure at the rate of 70/45 mm Hg, bradycardia. Additional studies revealed the reduced concentration of aldosterone and cortisol in blood, decreased excretion of 17-ketosteroids and 17-oxyketosteroids in the urine, hyponatremia, chloropenia, hypokalemia. What therapeutic measures are required?

Explanation

This patient presents with decreased aldosterone (mineralocorticoid); cortisol (glucocorticoid); hyponatremia; hypotension - these are signs of adrenal insufficiency because aldosterone and cortisol are primarily produced in the Adrenal gland. 

Adrenal crisis, also termed acute adrenal insufficiency is an acute life-threatening condition. It is considered one of the endocrine emergencies precipitated by an internal or external process in the setting of known or unknown lack of production of the adrenal hormone cortisol, the major glucocorticoid.

The adrenal gland's main function is to produce both mineralocorticoids and glucocorticoids. There are other hormones secreted by the adrenal glands including catecholamines and precursors of sex hormones. The adrenocorticotropic hormone (ACTH) is produced by the pituitary gland and its main function is to stimulate the cortisol release from the adrenal glands.

The main function of aldosterone is sodium retention and potassium secretion. The cortisol promotes gluconeogenesis, increases sensitivity to catecholamines, and regulates the immune system. If a patient with primary insufficiency has an adrenal crisis, they may be found to be hyponatremic in addition to having hypoglycemia and hypotension due to both aldosterone and cortisol deficiency, respectively. They also present with hyperpigmentation as a result of increased ACTH levels.

 

It is now clear that the best treatment is to administer hormones that have low levels - glucocorticoids (cortisol); mineralocorticoids (aldosterone) and a diet with high content of cooking salt to correct hyponatremia.

144. A 23-year-old female patient has a mental disease since the age of 18, the course of disease has no remission periods. At a hospital the patient mostly presents with non-purposeful foolish excitation: she makes stereotypic grimaces, exposed, masturbating in front of a loud laugh, repeating the stereotypical abusive shouts. The patient should be assigned:

Explanation

Neuroleptics, also known as antipsychotic medications, are used to treat and manage symptoms of many psychiatric disorders. They fall into two classes: first-generation or "typical" antipsychotics and second-generation or "atypical" antipsychotics." They are Dopamine D2 receptor antagonists. Antipsychotics are drugs that have a specific sedative effect, and which improve the attitude and calm the behavior of psychotic patients.

Typical antipsychotics: Haloperidol, Chlorpromazine

Atypical antipsychotics: Risperidone, Clozapine, Aripiprazole

This patient is currently having a manic episode and it's best to use a Neuroleptic (antipsychotic) with sedative effect to calm the patient down.

A manic episode requires 3 or more of the following:

  • Decreased need for sleep

  • Distractibility

  • Impulsivity/Indiscretion: seeks pleasure without regard to consequences including sexual pleasure

  • Talkativeness or pressured speech

  • Grandiosity: inflated self esteem

  • Flight of ideas: racing thoughts

  • Increased goal oriented activity/psychomotor agitation

Antidepressants are used for Major Depressive Disorders; Tranquilizers are used for General Anxiety Disorder or Panic attacks. Mood stabilizers like Lithium, Valproic acid, Lamotrigine, Carbamazepine are commonly used in Bipolar disorders. Nootropics are used to enhance mental function not mania.

 
145. A young woman with seborrhea adiposa has numerous non-itchy light brown and white spots with clear outlines and defurfuration on the torso and shoulder skin. What is the provisional diagnosis?

Explanation

The above description is typical for Tinea (pityriasis) versicolor which is caused by malassezia spp. A yeast like fungus. It is characterised by areas of hypopigmentation ‘white spots’. Hyperpigmentation can also occur due to inflammatory response - ‘areas of brown spots .’ It differs from dermatophytes because it is less pruritic (itchy).

Basically, all the other options listed here are itchy, while Pityriasis versicolor is non-itchy.

 

Pityriasis rosea is a viral rash that resembles small oval red patches and are very itchy. Seborrheic dermatitis is a common skin disease that causes an itchy rash with flaky scales and mainly affects the scalp, face and trunk. Vitiligo is a condition that is associated with pathological destruction of melanocytes.

146. 10 days after birth a newborn developed a sudden fever up to 38, 1oC. Objectively: the skin in the region of navel, abdomen and chest is erythematous; there are multiple peasnized blisters with no infiltration at the base; single bright red moist erosions with epidermal fragments on the periphery. What is your provisional diagnosis?

Explanation

Epidemic pemphigus of newborn or Pemphigus Neonatorum is a disease of the newborn, in which vesicles or blebs, usually flat on the top, appear with or without fever, on a perfectly normal or reddened skin with pin-head size vesicle elevation, which rapidly develops into a large bleb. As the disease progresses, relapses may occur, yet the infant may recover promptly after a single outbreak of but few blebs. In some cases these blebs show a marked tendency to peripheral extension, which finally dry, leaving a thin brown crust. Staphylococcus aureus have been implicated as the causative agent.

 

The distinction between Pemphigus and syphilis in the newborn may be determined by the localization of the eruption. Syphilis will be localized to the palms of the hands and soles of the feet, as well as by the associating symptoms of syphilis. Impetigo commonly presents with honey crusted vesicles and papules. Atopic dermatitis is an allergic reaction and patient most often have other allergic conditions or family history of asthma, allergic rhinitis, eczema etc.

147. An emergency physician arrived to provide medical care for a hangman taken out of the loop by his relatives. The doctor revealed no pulse in the carotid arteries, absence of consciousness, spontaneous breathing and corneal reflexes; cadaver spots on the back and posterior parts of extremities. A person can be declared dead if the following sign is present:

Explanation

Forensic pathologists are essentially required to estimate the time since death (TSD) to assist in death investigation. The early post-mortem phase is most frequently estimated using the classical triad of post-mortem changes – rigor mortis, algor mortis and livor mortis. 

Algor Mortis: Humans are warm-blooded organisms, which means that we maintain a constant internal temperature, regardless of the outside environment. The brain is our thermostat and the circulatory system is the main heat dissipator. However, within seconds of death, the brain cells begin to die and the heart stops pumping blood. Without the brain and and the blood distributing heat, the corpse eventually starts to match the outside temperature.

Rigor mortis is the post-mortem stiffening of muscles, caused by the depletion of adenosine triphosphate (ATP) from the muscles.

Livor Mortis: This is the final stage of death. When the heart stops beating, the blood is now at the mercy of gravity. It tends to collect at certain parts of the body. Depending on the position of the body, these parts would vary. For instance, if the person was flat on their back when they died, the blood would collect in the parts that are touching the ground (just as in this patient - cadaver spots were on the back and posterior parts of the extremities). If the person was hanging, it would collect in their fingertips, toes, and earlobes. Lividity starts with the skin where the blood has settled developing a bright red color. After a few hours, the color changes from red to bluish-purple. The bluish coloring of the skin is called livor mortis or cadaver spots.

Absence of spontaneous breathing, pulselessness or unconsciousness can be as a result of collapse (syncope) and not outright death. Absence of corneal reflexes is very possible and can be diagnosed in patients that are alive but with cranial nerve disorders (especially cranial nerve V and VII).

 
148. A 40-year-old patient is registered in a narcological dispensary. Somatically: skin is dramatically hyperemic, sclera are injected, hyperhidrosis is present. AP-140/100 mm Hg, heart rate - 100/min. Mental state: autopsychic orientation is intact, allopsychic orientation is distorted. The patient presents with motor anxiety. There is a look of fear on his face. He refuses to talk about his problems and asks to release him immediately, because he \\\"may be killed.\\\"This state developed a day after a regular drinking bout. What is your provisional diagnosis?

Explanation

Delirium tremens also known as Alcohol withdrawal delirium is the most severe form of alcohol withdrawal symptoms. It is characterised by an altered mental status and hyperactivity of the autonomic nervous system. Remember from the question stem, this started a day after one of his drinking bouts. 

Delirium tremens occurs in chronic alcohol abusers who abruptly discontinue alcohol use, often as early as 24 - 48 hours. Alcohol acts as a central nervous system depressant. It enhances the effect of inhibitory neurotransmitters while down-regulating excitatory neurotransmitters. Abrupt cessation of alcohol causes a decrease in the inhibitory actions of GABA neurotransmitter resulting in overactivity of the central nervous system. 

 

The overactivity seen with alcohol withdrawal manifests as altered mental status, hallucinations (“may be killed”), anxiety, seizures, tremors, psychomotor agitation, insomnia etc.

149. A 35-year-old female patient has gained 20 kg weight within a year with the normal diet. She complains of chill, sleepiness, shortness of breath. The patient’s mother and sister are corpulent. Objectively: height - 160 cm, weight - 92 kg, BMI - 35,9. Obesity is uniform, there are no striae. The face is amimic. The skin is dry. The tongue is thickened. Heart sounds are muffled. HR- 56/min, AP-140/100 mm Hg. The patient has constipations, amenorrhea for 5 months. TSH-28 mkME/l (normal rate - 0,32-5). Craniogram shows no pathology. What is the etiology of obesity

Explanation

Your thyroid produces thyroid hormone, which controls many activities in your body, including how fast you burn calories and how fast your heart beats. Diseases of the thyroid cause it to make either too much or too little of the hormone. Depending on how much or how little hormone your thyroid makes, you may often feel restless or tired, or you may lose or gain weight. Women are more likely than men to have thyroid diseases, especially right after pregnancy and after menopause.

Hypothyroidism is when your thyroid does not make enough thyroid hormones. It is also called underactive thyroid. This slows down many of your body's functions, like your metabolism. Signs and symptoms include the following: Feeling cold when other people do not; Constipation; weight gain, even though you are not eating more food; Feeling sad or depressed; Feeling very tired; Pale, dry skin; Dry, thinning hair; Slow heart rate (bradycardia); puffy face;  hoarse voice; abnormal menstrual bleeding etc.

Note that the craniogram shows no pathology so we can rule out Hypothalamic-pituitary. Hypo-ovarian can present with amenorrhea, vaginal dryness, monophasic cycles etc but not with bradycardia or weight gain. Absence of abdominal striaes or skin hyperpigmentation and sometimes diabetes - rules out the option of cushing’s disease (hypercorticoidism). 

 

The increased TSH levels seen in this patient implies an activation of the feedback mechanism signalling  the body to produce more thyroid hormones; T3 and T4.

150. A 26-year-old patient consulted a doctor abut sore throat, fever up to 38, 2oC. A week ago, the patient had angina, didn’t follow medical recommendations. On examination, the patient had forced position of his head, trismus of chewing muscles. Left peritonsillar region is markedly hyperemic, swollen. What is the provisional diagnosis?

Explanation

Extension of a tonsil infection can take place in the surrounding tissue and is referred to as  a Peritonsillar abscess or quinsy. The 26 year old man had previously had angina (acute tonsillitis) but didn’t follow medical advice. This complication of tonsillitis is usually caused by a B-hemolytic streptococcal infection and are characterised by an extremely sore throat and high fever (as seen in the patient) . 

A peritonsillar abscess forms in the tissues of the throat next to one of the tonsils. An abscess is a collection of pus that forms near an area of infected skin or other soft tissue. The abscess can cause pain, swelling, and, if severe, blockage of the throat. If the throat is blocked, swallowing, speaking, and even breathing become difficult.

 

The key sign that distinguishes quinsy/peritonsillar abscess from ordinary tonsillitis is the presentation of trismus. Trismus is usually absent in any form of tonsillitis but would be seen in a Peritonsillar abscess.

151. A 77-year-old patient complains of inability to urinate, bursting pain above the pubis. The patient developed acute condition 12 hours ago. Objectively: full urinary bladder is palpable above the pubis. Rectal prostate is enlarged, dense and elastic, well-defined, with no nodes. Interlobular sulcus is distinct. Ultrasonography results: prostate volume is 120 cm3, it projects into the bladder cavity, has homogeneous parenchyma. Prostate-specific antigen rate is of 5 ng/ml. What is the most likely disease that caused acute urinary retention?

Explanation

Benign Prostate Hyperplasia (BPH) is a non carcinogenic pathology that occurs in men usually above 40 years of age. It presents with symptoms similar to Prostate cancer. These include painful urination (dysuria), acute urinary retention, urinary frequency, hesitancy, dribbling, and frequent nighttime urination (nocturia). To differentiate between these two pathologies, an objective examination is carried out - a digital rectal examination or prostate biopsy. In this case, on digital rectal examination, the prostate is dense, enlarged, elastic and well defined with no nodes - these findings support the initial diagnosis of BPH because on digital rectal examination, prostatic carcinoma will not be well defined and will present with nodes. To further prove this diagnosis, the Prostate specific antigen is measured (normal <4). A marginal increase is going to support the diagnosis of BPH while PSA of 10 and above will indicate prostate cancer.
152. A 10 week pregnant woman was admitted to a hospital for recurrent pain in the lower abdomen, bloody discharges from the genital tracts. The problems turned up after ARVI. The woman was registered for antenatal care. Speculum examination revealed cyanosis of vaginal mucosa, clean cervix, open cervical canal discharging blood and blood clots; the lower pole of the gestational sac was visible. What tactics should be chosen?

Explanation

This is a case of retained product of conception which probably is an outcome of a missed abortion. Retained product of conception is a very common gynecological condition that you will definitely experience in your everyday Gynecological practice. She had acute respiratory viral infection (ARVI) which must have caused the abortion/miscarriage and presented with bloody discharge along with blood clots with no evidence of a live fetus, just the gestational sac was visible. In such conditions, a curettage or Manual Vacuum Aspiration (MVA) is done to get out the retained product of conception and the bleeding will stop.

Indication for curettage includes Abnormal uterine bleeding: irregular bleeding, menorrhagia, suspected malignant or premalignant condition, Retained material in the endometrial cavity, Evaluation of intracavitary findings from imaging procedures (abnormal endometrial appearance due to suspected polyps or fibroids),

 

Evaluation and removal of retained fluid from the endometrial cavity (hematometra, pyometra) in conjunction with evaluating the endometrial cavity and relieving cervical stenosis etc.

153. A patient with fibromyoma of uterus sized up to 8-9 weeks of pregnancy consulted a gynaecologist about acute pain in the lower abdomen. Examination revealed pronounced positive symptoms of peritoneal irritation, high leukocytosis. Vaginal examination revealed that the uterus was enlarged corresponding to 9 weeks of pregnancy due to the fibromatous nodes, one of which was mobile and extremely painful. Appendages were not palpable. There were moderate mucous discharges. What is the optimal treatment tactics?

Explanation

From the vaginal examination, we observe the presence of a very large fibroid mass that causes the enlargement of the uterus and is responsible for the severe pain the woman is experiencing and the mucous discharge. Coupled with the positive symptom of peritoneal  irritation, this patient is in need of an urgent surgical procedure.

Uterine fibroids are noncancerous growths that grow in the wall of the uterus. When fibroids cause heavy bleeding or painful symptoms, and other treatments are ineffective, a doctor may recommend surgery. People with asymptomatic fibroids do not require surgery or other treatments. However, other people experience severe abdominal pain, pressure, bloating, pain during sex, frequent urination, and heavy or painful periods. These individuals may require surgery.

 

Myomectomy is a surgical procedure that removes fibroids. Depending on the location of these growths, a surgeon may also have to remove other tissue in the process. The traditional technique is quite invasive as it uses a relatively large cut (Laparotomy). This incision may go from the bellybutton to the bikini line or run horizontally along the bikini line. Some surgeons also perform laparoscopic surgeries, which use smaller incisions but require more skill. However, in this case, a fibroid as big as 9 weeks gestation will require a big incision and laparoscopic surgery will not be able to get the fibroid out.

154. A 40-year-old female patient complains of having a bulge on the anterior surface of neck for 5 years. Objectively: Ps- 72 bpm, arterial pressure - 110/70mm Hg, in the right lobe of thyroid gland palpation reveals a mobile 4x2 cm node, the left lobe is not palpable, the basal metabolic rate is 6%. What is the most likely diagnosis?

Explanation

 

Notice that the Blood pressure and heart rate are normal which indicates no case of hyperthyroidism or hypothyroidism. Recall that a goiter occurring in a hyperthyroid state is a Hyperthyroid goiter; one that occurs in a hypothyroid state is a hypothyroid goiter while a goiter that occurs in a normal physiologic state as it is in this patient is an Euthyroid goiter. This is a nodular form because on palpation, a mobile node was palpated.

155. A multigravida at 39 weeks of gestation has been delivered to a hospital having a regular labour activity for 8 hours, the waters burst an hour ago. She complains of headache, seeing spots. AP is of 180/100mm Hg. Urine test results: protein - 3,3g/l, hyaline cylinders. Fetal heart rate is 140/min, rhythmical. Vaginal examination reveals complete crevical dilatation, the fetal head is on the pelvic floor, sagittal suture is in line with obstetric conjugate, the occipital fontanel is under the pubis. What is the optimal tactics of labour management?

Explanation

An outlet forceps is used in assisting delivery when there is full cervical dilation with the fetal head reaching the perineal floor and the scalp is visible between contractions. This type of assisted delivery is performed only when the fetal head is in a straight forward or backward vertex position or in slight rotation (less than 45 degrees to the right or left) from one of these positions.

The patient in question is manifesting signs and symptoms of pre-eclampsia already: proteinuria 3.3g/L; elevated blood pressure 180/100mmHg; headache and seeing spots; pregnancy is already beyond 20 weeks of gestation. The fetus is still stable judging from the fetal heart rate of 140beats per min. So, with a full cervical dilation, the best option is to assist the delivery using an outlet forceps.

Full cervical dilatation indicates completion of the first stage of labor and if the head of the fetus has descended, then an assisted vaginal delivery can be pursued without much risk to both the mother and fetus.

With a full cervical dilatation (completion of the first stage of labor) and in the absence of fetal distress, a cesarean section will be unnecessary. Cavity forceps and Vacuum extraction can be used when the head of the fetus is still high up in the uterus (i.e. yet to descend) or in the case of a retained second twin in multiple gestation. 

 

We can’t go for the option of conservative labor management in this condition because the patient is already showing signs of pre-eclampsia and delivery should be done promptly.

156. After a contact with chemicals a plant worker has suddenly developed stridor, voice hoarseness, barking cough, progressing dyspnea. Objective examination reveals acrocyanosis. What is your provisional diagnosis?

Explanation

Laryngeal edema is a common complication in patients that have allergic predisposition. Symptoms of stridor, hoarseness, and tachypnea developed within 24 hours after contact with the offending agent which could be a bee sting, chemicals, animal dander, dust, pollen etc and subside within another 24 hours after inhalation therapy with heated aerosol mist. 

 

The contact with chemicals led to the edematic reaction in the worker’s larynx. This is a possible irritation caused by chemical agents. Recall that the larynx is the voicebox; this is why the edematic reaction presents with symptoms such as stridor, voice hoarseness, barking cough etc

157. A 58-year-old patient complains of general weakness, loss of 10 kg of weight within 1,5 months, progressive pain in the lumbar region, increased blood pressure up to 220/160 mm Hg, subfebrile temperature. Objectively: in the right hypochondrium palpation reveals a formation with uneven surface and low mobility; veins of the spermatic cord and scrotum are dilated. Blood test results: Hb- 86 g/l, ESR- 44 mm/h. Urine test results: specific gravity - 1020, protein - 0,99 g/l, RBCs - cover the whole field of vision, WBCs - 4-6 in the field of vision. What is the provisional diagnosis?

Explanation

The fact that the patient lost a lot of weight in a short period of time in the absence of any other pathology should signal a carcinogenic process. Key diagnostic features of this pathology include; blood in urine, presence of flank mass and pain accompanied with weight loss and high blood pressure. Note, most renal pathologies present with elevated blood pressure due to the Renin-Angiotensin-Aldosterone System (RAAS). The dilated veins of the spermatic cord and scrotum (varices) is due to disrupted venous blood flow caused by increased pressure from the obstruction caused by the tumour.

A patient with urolithiasis will also experience colicky (more like an attack with relieve period in between) flank pain, change in quantity of urine and can be differentiated by results from ultrasound which shows an obstruction in the ureter. 

 

Glomerulonephritis often follows an upper respiratory tract infection 2-3weeks prior to presentation and can also present with a nephrotic syndrome (increased proteinuria accompanied by presence of edema). Pyelonephritis is inflammation of the kidney parenchyma which can be as a result of infection via an ascending or hematogenous pathway.

158. A 58-year-old patient complains of a headache in the occipital region, nausea,choking, opplotentes. The presentations appeared after a physical exertion. Objectively: the patient is excited. Face is hyperemic. Skin is pale. Heart sounds are regular, the 2nd aortic sound is accentuated. AP- 240/120 mm Hg, HR-92/min. Auscultation reveals some fine moist rales in the lower parts of the lungs. Liver is not enlarged. ECG shows signs of hypertrophy and left ventricular overload. What is the most likely diagnosis?

Explanation

The patient presented with a blood pressure of 240/120mmHg which is markedly elevated. This increased blood pressure already led to the development of hypertrophy and left ventricular overload as stated in the question stem which are common complications associated with extremely high blood pressure. Initially, cardiac hypertrophies are usually compensatory mechanisms but if the blood pressure is not promptly and properly managed, it can become a full blown cardiac failure.

It is clear that this is a case of Complicated Hypertensive crisis. It is complicated with hypertrophy and pulmonary edema. The pulmonary edema is evidenced by the objective findings on auscultation of the lungs that revealed fine moist rales in the lower parts of the lungs.

 

This is not a case of acute myocardial infarction. Myocardial infarctions are often associated with severe substernal chest pain or tightness. There was no respiratory distress of any sort, so we can rule out both bronchial asthma and pneumonia.

159. A patient complains of being unable to get pregnant for 5 years. A complete clinical examination gave the following results: hormonal function is not impaired, urogenital infection hasn’t been found, on hysterosalpingography both tubes were filled with the contrast medium up to the isthmic segment, abdominal contrast was not visualized. The patient’s husband is healthy. What tactics will be most effective?

Explanation

In vitro fertilization (IVF) is a complex series of procedures used to help with fertility or prevent genetic problems and assist with the conception of a child. During IVF, mature eggs are collected (retrieved) from ovaries and fertilized by sperm in a lab. Then the fertilized egg (embryo) or eggs (embryos) are transferred to a uterus. One full cycle of IVF takes about three weeks. IVF is the most effective form of assisted reproductive technology. The procedure can be done using your own eggs and your partner's sperm. Or IVF may involve eggs, sperm or embryos from a known or anonymous donor. 

IVF is the best option if the woman has Fallopian tube damage or blockage. Fallopian tube damage or blockage makes it difficult for an egg to be fertilized or for an embryo to travel to the uterus. This is the case with this patient. The contrast medium got to the isthmic segment of the fallopian tube and wasn’t visualized in the abdominal cavity. This clearly indicates a blockage somewhere beyond the isthmic segment.

 

With this pathology, insemination with husband’s semen will not yield any result because the oocyte can’t travel to the uterus due to the blockage in the fallopian tube. In artificial insemination, sperm is placed in the uterus and conception happens otherwise normally. IVF involves combining eggs and sperm outside the body in a laboratory. Once an embryo or embryos form, they are then placed in the uterus. Research shows that IVF is just as effective as the ICSI (Intracytoplasmic sperm injection) procedure, where sperm is injected directly into an egg, when there is no male infertility factor.

160. Six months ago, a 5-year-old child was operated for CHD. For the last 3 weeks he has complained of fever, heart pain, aching muscles and bones. Examination results: \\\"white-coffee\\\"skin colour, auscultation revealed systolic murmur in the region of heart along with a noise in the III-IV intercostal space. Examination of fingertips revealed Janeway lesions. What is your provisional diagnosis?

Explanation

Infective endocarditis (IE) is caused by damage to the endocardium of the heart followed by microbial, usually bacterial, colonization. It is an infection in the heart valves or endocardium. The endocardium is the lining of the interior surfaces of the chambers of the heart. Infective endocarditis disproportionately affects those with underlying structural heart disease and is increasingly associated with healthcare contact (the child was operated for congestive heart disease - CHD), particularly in patients who have intravascular prosthetic material. 

 

Symptoms include: fever (most common); Janeway lesions (small, painless, erythematous lesions on palm or sole); new cardiac murmur; Roth spots (round white spots on retina surrounded by hemorrhage); Osler nodes (raised lesions on finger or toe pads due to immune complex deposition); and splinter hemorrhages on nail bed.

161. A 45-year-old female patient has worked as a painter for 14 years. Contacts with synthetic paint result in face skin redness, swelling, intense itching, oozing lesions. The symptoms disappear after the exposure to chemical agents, but recur even at the smell of paint. The symptom intensity progresses with relapses. Make a provisional diagnosis:

Explanation

Eczema (dermatitis ) is a chronic inflammatory disease of the skin characterized by the presence of red, itchy, dry scaly rashes. Occupational or professional  eczema (dermatitis) is gotten from contact with certain chemicals in the course of working (in this case due to contact with the chemicals in paint ). 

 

Allergic dermatitis is a form of contact dermatitis that results from immune reactions towards certain irritants. This term can be used if it’s unrelated to the patient’s work.

162. In a cold weather, the emergency room admitted a patient pulled out of the open water. There was no respiratory contact with the water. The patient is excited, pale, complains of pain, numbness of lands and feet, cold shiver. Breathing raten is 22/min, AP- 120/90 mm Hg, Ps- 110/min, rectal temperature is 34, 5oC. What kind of warming is indicated for this patient?

Explanation

Hypothermia is a severe condition in which the body temperature drops to an abnormally low level. It occurs when the body is unable to produce enough heat to counter the heat that it is losing. Under healthy conditions, the body maintains a relatively stable temperature of around 98.6˚F or 37˚C. 

The following techniques can help treat hypothermia:

Passive external rewarming: This uses the individual’s heat-generating ability. It involves removing their cold, wet clothing, ideally replacing it with adequately insulated, dry clothing, and moving them to a warm environment. 

Active external rewarming: This involves applying warming devices, such as hot-water bottles or warmed forced air, externally to truncal areas of the body. For example, the individual could hold a hot-water bottle under each arm.

Active core rewarming: This uses warmed, intravenous fluids to irrigate body cavities, including the thorax, peritoneum, stomach, and bladder. Other options include getting the individual to inhale warm, humidified air, or applying extracorporeal rewarming by using a heart-lung machine.

Do not give a person alcohol if they have signs of hypothermia, and avoid giving any drinks to an unconscious person.

 

From the vitals given, this patient is fairly stable except for the tachycardia (110bpm) and low body temperature, therefore passive warming will suffice. The patient was pulled out of the open water in a cold weather so removing the wet clothing, and replacing it with adequately insulated, dry clothing, and moving them to a warm environment will keep the patient warm.

163. Survey radiograph of a 52-year-old worker of an agglomeration plant (28 years of experience, the concentration of metal dust is 22-37 mg/m3) shows mildly pronounced interstitial fibrosis with diffused contrast well-defined small nodular shadows. The patient has no complaints. Pulmonary function is not compromised. What is the provisional diagnosis?

Explanation

 

Agglomeration plants are factories that work mainly with Iron ore materials together with other products, refining them into finished goods. This  patient has an accumulation of iron dust in his system as a result of long term exposure over the years. Pneumoconiosis is a restrictive lung disease  caused by the inhalation of dust leading to fibrosis. Depending on the type of dust inhaled, different types exist. They include; Siderosis (iron ore), byssinosis (cotton), anthracosis (coal), Asbestosis (asbestos), silicosis (silica dust).

164. A week ago a 65-year-old patient suffered an acute myocardial infarction, his general condition deteriorated: he complains of dyspnea at rest, pronounced weakness. Objectively: edema of the lower extremities, ascites is present. Heart borders are extended, paradoxical pulse is 2 cm displaced from the apex beat to the left. What is the most likely diagnosis?

Explanation

One of the most common complications occurring post-infarction is a ventricular aneurysm. Almost 85% to 90% of the ventricular aneurysms occur in the setting of acute anterior wall myocardial infarction. Left ventricular aneurysm is defined as a localized area of myocardium with abnormal outward bulging and deformation during systole and diastole. The natural course leading to the formation of a ventricular aneurysm involves a full-thickness (transmural) infarct that has been replaced by fibrous tissue. This inert portion cannot take part in the contraction and herniates outward during systole.

A week before presenting to the hospital, he had an acute myocardial infarction. A true left ventricular aneurysm following acute myocardial infarction can occur as early as within 48 hours or two weeks post-infarction. 

Common symptoms associated with the ventricular aneurysm include: Fatigue; Shortness of breath (dyspnea); Chest pain; Palpitations; Syncope; Fluid retention causing swelling of ankles, feet, or abdomen (ascites); Stroke; Limb or visceral ischemia.

 

A recurrent myocardial infarction will present with the classic substernal chest pain or tightness. Acute pericarditis will produce chest pain as well and friction rub between the pericardial sheets. Pulmonary embolism is usually a complication of deep vein thrombosis (DVT).

165. A 64-year-old patient has been referred to planned hospitalization for general weakness, poor appetite, progressive jaundice which appeared over 3 weeks ago and wasn’t accompanied by pain syndrome. Objectively: body temperature is at the rate of 36, 8oC, Ps-78/min, abdomen is soft and painless, the symptoms of peritoneal irritation are present, palpation reveals a dramatically enlarged, tense gallbladder. What disease are these symptoms typical for?

Explanation

The patient in question presents with obstructive/mechanical/post-hepatic jaundice. The gallbladder is enlarged and tense and the patient also presented with general weakness, poor appetite plus progressive jaundice.

Obstructive jaundice is a common clinical manifestation of pancreatic cancers, especially in patients with malignant tumors of the head of the pancreas. Obstructive Jaundice is a common surgical problem that occurs when there is an obstruction to the passage of conjugated bilirubin from liver cells to intestine.

Cancer of the head of the pancreas appears near the common bile duct. From an early stage, they tend to compress this duct leading to an obstruction in bile flow (causing an obstructive jaundice).

Apart from Cancer of the head of pancreas, other causes of obstructive jaundice include: biliary stricture (narrowing of the bile duct); cholangitis (infection or inflammation of the common bile duct); cholelithiasis (gallstones); cysts of the bile duct; parasitic infection etc.

 

Note that Cholecystitis usually presents with pain in the right upper quadrant and signs of systemic infection (pyrexia, raised leukocyte count, raised C-reactive protein). The above patient presents with none of these symptoms thereby ruling out the options of cholecystitis.

166. A 22-year-old vegetarian patient with signs of malnutrition consulted a doctor about smell and taste distortion, angular stomatitis. Objectively: expressively blue sclerae. The patient was diagnosed with iron deficiency anemia. What is the dominating clinical syndrome?

Explanation

Vegetarian diets are dietary patterns that are devoid of all flesh foods but may include egg or dairy products. Vegetarians have lower incidences of several health conditions including ischemic heart disease, total incidence of cancer, and type 2 diabetes. Despite these advantages, vegetarians also are at a high risk of a deficiency of some nutrients, such as vitamin B12 and iron. 

This patient with malnutrition also complained about smell and taste distortion. Loss of smell and/or taste has been linked to inadequate nutritional intake, reduced social pleasure, and decreased psychological well-being.

Anemia is defined by reduced hemoglobin level or the erythrocytes volume in the circulation. Sideropenic anemia is a hypochromic, microcytic anemia caused by insufficient iron level in the body. This is the most common anemia. 

Patients with Iron deficiency often manifest Pica. Pica is an unusual craving for and ingestion of either edible or inedible substances. It seems to be strongly associated with iron deficiency anemia, and in the majority of cases the unusual eating and chewing behavior disappears upon iron supplementation.

 

Patients with the diagnosis of sideropenic anemia are recommended to take iron supplements. By changing eating habits we can maximize iron input into our organism and prevent sideropenic anemia.

167. A 13-year-old girl was admitted to the gynecological department with heavy bleeding, which appeared after a long delay of menstruation. Shortly before, the girl suffered a serious psychotrauma. Her menarche occurred at the age of 11, she has a 30-day cycle with 5 to 6 days of moderate, painless bleeding. The patient is somatically healthy, of normosthenic constitution with height of 160 cm, weight of 42 kg. The patient is pale. Rectoabdominal examination revealed that the uterus was of normal size and consistency, anteflexio-versio, the appendages were not changed. What is the most likely diagnosis?

Explanation

 

Menstrual cycles are often irregular during adolescence, particularly the interval from the first cycle to the second cycle. Most females bleed for 2–7 days during their first menses. Immaturity of the hypothalamic–pituitary–ovarian axis during the early years after menarche often results in anovulation and cycles may be somewhat long; although short cycles of less than 20 days and long cycles of more than 45 days may occur. For the first few years after menstruation begins, long cycles are common. However, menstrual cycles tend to shorten and become more regular as you age. As for this patient her menstrual cycle is yet to stabilize, so it is a case of Female Juvenile bleeding.

168. It is planned to build a multidisciplinary hospital with 500 beds in a town. Specify the location of a polyclinic within the medical centre:

Explanation

169. A 48-year-old patient complains of weakness, subfebrile temperature, aching pain in the kidney region. These presentations turned up three months ago after hypothermia. Objectively: kidneys are painful on palpation, there is bilaterally positive Pasternatsky’s symptom. Urine test res: acid reaction, pronounced leukocyturia, microhematuria, minor proteinuria - 0,165-0,33 g/l. After the urine sample had been inoculated on conventional media, bacteriuria were not found. What research is most required in this case?

Explanation

There are some characteristics in a urine examination that suggest a diagnosis of renal Tuberculosis, such as acid pH, leukocyturia and/or hematuria, associated with negative urine culture for the usual bacteria that causes urinary tract infection (bacteria were not found on conventional media).

Points worthy of note from the Urine test result: acid reaction, leukocyturia and after inoculation on conventional media, bacteriuria were not found.

Now, Mycobacterium tuberculosis is acid fast due to the presence of Mycolic acid, so they will produce an acid reaction. There is increased lymphocytes (leukocyturia) with M.tuberculosis and viral infection (Neutrophils will increase with bacteria - Neutrophilia). Also, M. tuberculosis will not grow on conventional media and they stain poorly with Gram staining because their cell wall has high lipid content. To detect M. tuberculosis, Ziehl-Neelsen stain (Carbol fuchsin) is used instead.

Nechiporenko: This method helps to determine the amount of cellular elements (WBC, RBC and casts) in 1 ml of urine.

Zimnitsky's test characterize the concentrating and excretory ability of the kidney.

Daily proteinuria monitoring will give information about the extent of nephrotic syndrome or any other nephropathy like hypertensive nephropathy.

 

Isotope renography is an imaging technique that’s used to check kidney function.

170. A 22-year-old patient complains of amenorrhea for 8 months. Menarche occurred at the age of 12,5. Since the age of 18 the patient has a history of irregular menstruation. The patient is nulligravida. The mammary glands are developed properly, nipples discharge drops of milk when pressed. Gynecological study results: prolactin level is 2 times higher than normal. CT reveals a bulky formation with a diameter of 4 mm in the region of sella. What is the most likely diagnosis?

Explanation

Pituitary Gland produces: Growth hormone; Prolactin; Thyroid Stimulating Hormone (TSH); Adrenocorticotropic Hormone (ACTH); Gonadotropins (FSH, LH). A tumor in the pituitary gland can lead to overproduction of any of these hormones. However, these are the very common ones: 

-Growth Hormone: (Children - Gigantism; Adult - Acromegaly)

-TSH: Hyperthyroidism

-ACTH: Cushing disease

-Prolactin: Too much prolactin, a hormone that stimulates lactation and the secretion of progesterone, causes inappropriate secretion of breast milk, even in men. It can also cause osteoporosis, which is weakening of the bones; loss of sex drive; infertility; irregular menstrual cycles; and the inability to have an erection.

 

People with a pituitary gland tumor may experience the following symptoms or signs. Headaches; Vision problems (Bitemporal hemianopsia); Changes in menstrual cycles in women (amenorrhea); Erectile dysfunction, Infertility; Inappropriate breast growth or production of breast milk; Cushing’s syndrome; Acromegaly, the enlargement of the arms or legs and thickening of the skull and jaw caused by too much growth hormone.

171. A 38-year-old female patient complains about hot flashes and feeling of intense heat arising up to 5 times a day, headaches in the occipital region along with high blood pressure, palpitations, dizziness, fatigue, irritability, memory impairment. 6 months ago the patient underwent extirpation of the uterus with its appendages. What is the most likely diagnosis?

Explanation

6 months ago the patient underwent extirpation (surgical resection) of the Uterus and its appendage (ovaries and fallopian tubes). Castration means loss of the gonads - testicles, ovaries. The loss could be by any means be it chemical, surgical or otherwise.

With the absence of the ovaries, there will be lack of female reproductive hormones - estrogen and progesterone which is synthesized in the ovaries normally. 

The absence of Estrogen especially will now create an artificial menopausal state manifesting with irritability, fatigue, hot flashes and feeling of intense heat. These are the symptoms seen in menopause due to lack of estrogen but we can now see it in this patient because the ovaries have been removed.

 

In menopause, the ovaries are present but they no longer produce adequate estrogen. However, in this case, the ovaries are out so Estrogen is not being produced.

172. A 2-year-old child in a satisfactory condition periodically presents with moderate proteinuria, microhematuria. USI results: the left kidney is undetectable, the right one is enlarged, there are signs of double pyelocaliceal system. What study is required to specify the diagnosis?

Explanation

Intravenous urography (IVU), also referred to as intravenous pyelography (IVP) or excretory urography (EU), is a radiographic study of the renal parenchyma, pelvicalyceal system, ureters and the urinary bladder. Intravenous or Excretory Urography is an X-ray procedure that involves the intravenous administration of contrast material inorder to  verify and localize defects in the urinary system or upper urinary tract diseases.  

A micturating cystourethrogram (MCUG) is a scan that shows how well your child’s bladder works. It is used to show how the urinary bladder is emptying. 

 

Retrograde Urography/Pyelogram also uses a contrast agent for better visualization of the ureters and kidneys but note that: In intravenous pyelogram, the contrast dye is injected into  a vein while in retrograde pyelogram, it is injected directly into the ureters. Retrograde pyelogram is mostly done in cases where excretory urography does not give a clear image of the pathological area and in this case, excretory urography will give a better result as it involves an undetectable kidney.

Excretory Urography - One Absent KidneyExcretory Urography - Normal Kidney

173. Explosion of a tank with benzene at a chemical plant has killed and wounded a large number of people. There are over 50 victims with burns, mechanical traumas and intoxication. Specify the main elements of medical care and evacuation of population in this situation:

Explanation

In this case, A Triage system is to be followed. Triage is a procedure followed in a medical emergency. It involves the sorting of and allocation of treatment to patients and especially battle and disaster victims according to a system of priorities designed to maximize the number of survivors

 These individuals should be allocated into categories such as: Minor, Deceased, Immediate and delayed. Those in the immediate category require instant medical assistance. Lastly, all 50 victims should be evacuated from the site of explosion.
174. A 26-year-old patient with affective bipolar disorder has developed a condition manifested by mood improvement, behavioural and sexual hyperactivity, verbosity, active body language, reduced need for sleep. Which of the following drugs are most effective in this case?

Explanation

Neuroleptics, also known as antipsychotic medications, are used to treat and manage symptoms of many psychiatric disorders. They fall into two classes: first-generation or "typical" antipsychotics and second-generation or "atypical" antipsychotics." They are Dopamine D2 receptor antagonists. Antipsychotics are drugs that have a specific sedative effect, and which improve the attitude and calm the behavior of psychotic patients.

Typical antipsychotics: Haloperidol, Chlorpromazine

Atypical antipsychotics: Risperidone, Clozapine, Aripiprazole

Neuroleptic drugs are beneficial for the management of behavioral disorders. They are also associated with sedative effects, weight gain, or anticholinergic activity.

Bipolar disorder: a manic episode and depressed mood. For treatment of Bipolar disorders: Lithium, Valproic acid, Lamotrigine, Carbamazepine, and Atypical antipsychotics. This patient is currently having a manic episode and it's best to use a Neuroleptic (antipsychotic) with sedative effect to calm the patient down and to sleep.

A manic episode requires 3 or more of the following:

  • Decreased need for sleep

  • Distractibility

  • Impulsivity/Indiscretion: seeks pleasure without regard to consequences including sexual pleasure

  • Talkativeness or pressured speech

  • Grandiosity: inflated self esteem

  • Flight of ideas: racing thoughts

  • Increased goal oriented activity/psychomotor agitation

 

Antidepressants are used for Major Depressive Disorders; Tranquilizers are used for General Anxiety Disorder or Panic attacks.

175. An emergency doctor has diagnosed a 32-year-old woman with generalized convulsive status epilepticus. The deterioration in the patient’s condition is caused by a sudden gap in the epilepsy treatment. Specify the doctor’s further tactics:

Explanation

Status epilepticus is said to occur when a seizure lasts too long or when seizures occur close together and the person doesn't recover between seizures. Over the last several decades, the length of seizure that is considered as status epilepticus has shortened. Years ago, a seizure needed to last longer than 30 minutes to be considered status epilepticus. In the last few years, it is now defined as any seizure greater than 5 minutes. This makes sense because most seizures do not last longer than 2 minutes. Very long seizures (i.e., status epilepticus) are dangerous and even increase the chance of death.

Status epilepticus requires emergency treatment by trained medical personnel in a hospital setting. This situation can be life-threatening and getting treatment started fast is vital.

 
  • Medical treatment needs to be started as soon as possible. Oxygen and other support for breathing, intravenous fluids (fluid given into a blood vessel), and emergency medications are needed.

  • Continuous EEG (electroencephalogram) monitoring may be needed to monitor the seizures and how a person responds to treatment.

  • At times, medicines called anesthetics are used in the hospital to put a person into a coma to stop the seizures. - This will definitely be done in the Intensive Care Unit (ICU).

176. A 19-year-old patient complains of dyspnea on exertion. He often has bronchitis and pneumonia. Since childhood, the patient presents with cardiac murmur. Auscultation revealed splitting of the II sound above the pulmonary artery, systolic murmur in 3 intercostal space at the left sternal border. ECG showed right bundle branch block. What is the provisional diagnosis?

Explanation

first heart sound - Mitral and Tricuspid valve

Second heart sound - Aortic and Pulmonic valve

Closure of these valves during cardiac cycle produces the sound. 

The second heart sound can have a physiologic splitting i.e. the aortic valve closes just before the pulmonic valve. This can occur when there is more blood flow through the pulmonic valve for example during inspiration there is increased blood return to the right side of the heart via the Superior vena cava and Inferior vena cava. This increased blood volume takes a longer time to pass through the pulmonic valve and this causes a slight delay - the aortic valve closes just before the pulmonic valve causing a physiologic splitting of S2.

Also, splitting of the second heart sound is commonly seen in Atrial Septal Defect, Pulmonic valve stenosis or right bundle branch block. In Atrial Septal Defect, there is a left to right shunt. Blood from the left atrium moves to the right atrium when they contract and this increases the blood volume in the right side of the heart. This increased blood volume takes longer to pass through the pulmonic valve. Thus, producing a split in the second heart sound.

 

Open Ductus Arteriosus will produce a continuous machine-like murmur throughout systole (heart contraction) and diastole (heart relaxation). Mitral insufficiency will produce a systolic murmur (holosystolic - throughout systole). Aortic stenosis will produce a bruit or murmur beyond the point of the stenosis. None of these other conditions produces a splitting of the second heart sound.

177. 20 minutes after a normal delivery at 39 weeks a puerpera had a single temperature rise up to 38oC. Objectively: the uterus is dense, located between the navel and the pubis, painless. Lochia are bloody, of small amount. Breasts are moderately soft and painless. What is the optimal tactics?

Explanation

This condition is optimal for immediate postpartum and does not require immediate actions to be taken. Uterus is painless and located between the navel and pubis is normal. Bloody lochia observed immediate postpartum is normal. Breast is also painless.

 

The only abnormality here is the slightly elevated body temperature of 38*C which might be reactive as a result of the stress, shouting and crying during labour. Therefore, follow-up is the best tactic to employ in this situation.

178. A 30-year-old patient got in a car accident. He is unconscious, pale, has thready pulse. In the middle third of the right thigh there is an extensive laceration with ongoing profuse external arterial bleeding. What urgent actions must be taken to save the life of the patient?

Explanation

The idea here is to Place the tourniquet between the injured vessel and the heart and the tourniquet should be over a bone, not on a joint. For arterial bleeding, the tourniquet should be above the wound which usually falls between the wound and the heart. And venous bleeding is usually below the wound.

Understanding the anatomy will help you to understand this better. Arteries take blood away from the heart, so for arterial bleeding, the blood is flowing from the heart to the wound, so you tie above it to stop the flow.

 

For Venous bleeding, Veins take blood to the heart, so you tie below the wound to stop the flow towards the wound (heart).

179. A 75-year-old male patient complains of slight pain in the right iliac region. The abdominal pain arose 6 days ago and was accompanied by nausea. Surgical examination revealed moist tongue, Ps- 76 bpm. AP- 130/80 mm Hg. Abdomen was soft, slightly painful in the right iliac region on deep palpation, the symptoms of the peritoneum irritation were doubtful. In blood: RBCs - 4, 0 · 1012/l, Hb- 135 g/l, WBCs - 9, 5 · 109/l, stab neutrophils - 5%, segmentonuclear - 52%, lymphocytes - 38%, monocytes - 5%, ESR - 20 mm/h. Specify the doctor’s further tactics:

Explanation

Definitive treatment for Acute Appendicitis is Surgical. Conservative management with medications to relieve pain or stop inflammatory processes will only produce temporary relief. 

ESR is elevated which is a sign of an inflammatory process. Right iliac fossa tenderness and right iliac fossa pain (Rebound tenderness, blumberg sign) are pointers to Acute Appendicitis.

Diagnosis of Appendicitis is made clinically, Ultrasound or abdominal X-ray will not give a definite diagnosis. When we say a diagnosis is clinical, it means the diagnosis is based on signs and symptoms elicited clinically. In this case, positive signs of appendicitis will give a better diagnosis than report of X-ray or Ultrasound. 

 

Appendiceal signs: Pain elicited in any of these signs indicates a positive test

Abdominal guarding

Rebound tenderness: To elicit the sign, gentle pressure is placed on the right iliac fossa of the abdomen and then the hand is lifted suddenly. A sudden increase in abdominal pain occurs when the examiner's hand is lifted. 

Rovsing's sign is elicited by pushing on the abdomen in the left lower quadrant as in most people the appendix is in the right lower quadrant. While this maneuver stretches the entire peritoneal lining, it only causes pain in any location where the peritoneum is irritating the muscle. In the case of appendicitis, the pain is felt in the right lower quadrant despite pressure being placed elsewhere.

 

Obturator sign: First the patient lies on his back with the right hip flexed at 90 degrees. The examiner then holds the patient's right ankle in his right hand. With his left hand, the examiner rotates the hip by pulling the right knee to and away from the patient's body.

 
180. A 55-year-old patient whose menstruation stopped 5 years ago complains of vaginal dryness, frequent and painful urination. Gynecologist revealed signs of atrophic colpitis. Urine analysis revealed no peculiarities. Which locally acting product will provide the proper therapeutic effect?

Explanation

Menopause is signalled by 12 months of complete cessation of menses. This patient is already 5 years into menopause. Menopause is characterized by significantly decreased female reproductive hormones especially Estrogen and Progesterone. Lack of Estrogen can manifest as vaginal dryness, osteoporosis and hot flashes. To remedy this situation, hormone replacement therapy can be utilized. Vaginal dryness is often treated with topical lubricants or oestrogen. Ovestin contains the female hormone - Estrogen and it can provide symptomatic relief in this condition.

 

Metronidazole, Tergynan, Meratin Combi, Dalacin are basically antibiotics used to treat gynecological infections. This patient, who is in menopause, is in need of hormone replacement and not antibiotics.

181. A 63-year-old patient with persistent atrial fibrillation complains of moderate dyspnea. Objectively: peripheral edemata are absent, vesicular respiration is present, heart rate - 72/min, AP- 140/90 mm Hg. What combination of drugs will be most useful in the secondary prevention of heart failure?

Explanation

Beta blockers (drugs ending with the suffix ‘lol’ e.g. Metoprolol, propranolol, labetalol) can be used as antiarrhythmic drugs (treat arrhythmias like Atrial fibrillation). They belong to class II of the antiarrhythmic medications. They can also be used in hypertensive conditions. Beta 1 receptors are present in the heart and when a Beta blocker is deplored, it can reduce the heart rate thereby reducing the cardiac output and contractility. They can also reduce the transmission of impulses through the AV node which makes them useful as antiarrhythmic agents.

Angiotensin Converting Enzyme Inhibitors (ACEI) inhibit the conversion of Angiotensin I to Angiotensin II thereby inhibiting the vasoconstrictive properties of Angiotensin II. This makes them very useful as antihypertensives. 

In this case, the question is interested in how to prevent heart failure and a combination of Beta blockers and ACE inhibitors are the most effective as they both reduce blood pressure and workload on the heart. They reduce both the preload and afterload of the heart which will prevent ventricular remodelling in the long run.

 

Cardiac glycosides tend to increase heart contractility and in the long run, this increase in workload can lead to hypertrophy. Diuretics can reduce the preload but not effectively reduce the afterload.

182. A 57-year-old patient had an attack of retrosternal pain that lasted more than 1,5 hours. Objectively: the patient is inert, adynamic, has pale skin, cold extremities, poor volume pulse, heart rate - 120/min, AP- 70/40 mm Hg. ECG shows ST elevation in leads II, III, aVF. What condition are these changes typical for?

Explanation

This patient with ST segment elevation in leads II, III, aVF on ECG that also had an attack of retrosternal pain is definitely a case of STEMI - ST elevation Myocardial Infarction. With a blood pressure of 70/40mmHg (severe hypotension), the condition has deteriorated to a case of shock and in this case, it has a cardiac origin, therefore, it is a cardiogenic shock.

 

There is no arrhythmia recorded on ECG (no Atrial flutter or fibrillation, ventricular tachycardia or fibrillation), so it will be wrong to tag this an Arrhythmogenic shock. Perforated gastric ulcers can present with excessive bleeding and even result in shock but the patient will also present with hematemesis or melena and epigastric pain. Acute pericarditis can present with retrosternal pain but not likely to have shock. Acute pancreatitis will present with diarrhea, indigestion, epigastric belt-like pain radiating to the back, and it is unlikely for a patient with pancreatitis to have retrosternal pain and shock with the listed ECG findings.

183. An 8-year-old boy has a 2-year history of blotchy itchy rash appearing after eating citrus fruit. The first eruption occurred at the age of 6 months after the introduction of juices to the baby’s diet. Father has a history of bronchial asthma, mother - that of allergic rhinitis. What is the most likely diagnosis?

Explanation

Chiefly, we can notice a trend of allergic diseases running in the family tree. Parental history of allergic conditions are strong risk factors for atopic dermatitis in the child.

Atopic dermatitis is an inflammatory skin disease that is characterised by itching, redness, and scaling of the skin, predominantly in the skin creases. This patient is allergic to citrus fruits. 

 

Atopic dermatitis is a systemic chronic allergic disease that occurs in people with an inherited predisposition to atopy and characterized by typical morphological changes of the skin with itching, lesions of the central and autonomic nervous system, endocrine and immune systems with hyper IgE.    Essential criteria for diagnosis: itch, rash elements`, typical morphology and localization (flexor surface of extremities in adults (lichenification), extensor surfaces and face in children (eczema)), chronic recurrent course, atopic diseases in personal and family anamnesis (bronchial asthma, allergic rhinitis).

184. On the 10th day postpartum a puerperant woman complains of pain and heaviness in the left mammary gland. Body temperature is 38, 8oC, Ps- 94 bpm. The left mammary gland is edematic, the supero-external quadrant of skin is hyperemic. Fluctuation symptom is absent. The nipples discharge drops of milk when pressed. What is a doctor’s further tactics?

Explanation

Mastitis is inflammation of the breast tissue and can be broken down into lactational and non-lactational mastitis. Lactational mastitis is the most common form of mastitis. Lactational mastitis, also known as puerperal mastitis, is typically due to prolonged engorgement of milk ducts, with infectious components from the entry of bacteria through skin breaks. Patients can develop a focal area of erythema, pain, and swelling, and can have associated systemic symptoms, including fever. This occurs most commonly in the first six weeks of breastfeeding but can occur at any time during lactation, with most cases falling off after 3 months. Lactational mastitis is most commonly caused by bacteria that colonize the skin, with Staphylococcus aureus being the most common, in this case an antibiotic may be administered to fight the infection.

The first line of treatment is self-help remedies, such as ensuring that the breast is drained properly during feeds by breastfeeding regularly or expressing the milk. And after a feed, gently express any leftover milk.

 

Inhibition of lactation will complicate the condition. This is not a case of breast abscess. An abscess will be associated with pus discharge from the nipple, not drops of milk. Use a hot compress on the breast will not be enough to resolve the condition since there is a probability of an ongoing infection.

185. A 25-year-old patient has been admitted to the hospital with the following problems: weakness, sweating, itching, weight loss, enlarged submandibular, cervical, axillary, inguinal lymph nodes. Objectively: hepatomegaly. Lymph node biopsy revealed giant Berezovsky Reed Sternberg cells, polymorphocellular granuloma made by lymphocytes, reticular cells, neutrophils, eosinophils, fibrous tissue, plasma cells. What is the most likely diagnosis?

Explanation

Lymphogranulomatosis, Hodgkin lymphoma or Hodgkin disease (HD) is a type of lymphoma and accounts for ~1% of all cancers. Hodgkin disease spreads contiguously and predictably along lymphatic pathways and is curable in ~90% of cases, depending on its stage and sub-type. Typical presentation is with painless lymphadenopathy. Systemic symptoms such as night sweats and weight loss may be present. 

 

The disease is characterized by the presence of Reed-Sternberg cells (considered to be a type of B cell). Reed-Sternberg cells are a classical finding diagnostic of Hodgkin lymphoma. They are giant, multinucleated cells with abundant pale cytoplasm. Reed-Sternberg cells are rare, making up <1% of lymphoid tissue, with the background consisting of lymphocytes, plasma cells, eosinophils and macrophages.

186. During self examination a 22 years old patient revealed a mammary tumour. Palpation revealed a firm, painless, mobile formation up to 2 cm, peripheral lymph nodes were not changed. USI results: in the superior external quadrant of the right mammary gland there was a big formation of increased echogenicity, sized 18x17 mm. The patient was provisionally diagnosed with fibroadenoma. What is a doctor’s further tactics?

Explanation

Fibroadenomas are typically present as firm, mobile, painless, and frequently multiple breast nodules. These tumors are common, benign breast tumors that usually affect women in second and third decades of life. Fibroadenomas are usually small and can be managed conservatively; however, a good percentage of these lesions will grow rapidly.

During pregnancy, fibroadenomas increase in size and may show lactational histologic changes. High concentrations of estrogen, progesterone, and prolactin promote the ductal growth and formation of tubuloalveolar structures. This may be a reason for the significant enlargement in this period.

Hence, it is recommended that it be removed prior to pregnancy.

187. A patient is 31 years old. Double contrast barium swallow revealed a filling defect on the posterior wall in the middle segment of esophagus. The defect looked like a well-defined oval 1,8x1,3 cm large. Mucosal folds adjacent to the defect were intact, peristalsis and elasticity of the walls remained unchanged. Digestive tract problems were absent. What is the provisional diagnosis?

Explanation

A filling defect was observed in the imaging technique used, which is a well defined oval mass indicating a tumor. Notice that mucosal walls are intact and wall peristalsis and elasticity are not affected- these rule out the options of achalasia cardia, Esophageal burns and barrett's esophagus ( because in these pathologies, one or more of the above listed is/are affected). The result from the x-ray shows a mass with clear oval borders thereby indicating a tumour.

 

Achalasia Cardia is a rare neurodegenerative disorder of esophagus resulting in defective peristalsis and impaired relaxation of lower esophageal sphincter. In Barrett's esophagus, the mucosal lining transforms from normal stratified squamous epithelium to simple columnar epithelium.

188. A 50-year-old patient has worked at a chemical plant for 15 years. His work involved using xylene solvent. The patient was hospitalized with suspected chronic intoxication. He was found to have anemic syndrome. What is the first priority measure of secondary anemia prevention?

Explanation

Prevention includes a wide range of activities — known as “interventions” — aimed at reducing risks or threats to health. Three categories of prevention: primary, secondary and tertiary.

Primary prevention aims to prevent disease or injury before it ever occurs. This is done by preventing exposures to hazards that cause disease or injury, altering unhealthy or unsafe behaviours that can lead to disease or injury, and increasing resistance to disease or injury should exposure occur. 

Secondary prevention aims to reduce the impact of a disease or injury that has already occurred. 

Tertiary prevention aims to soften the impact of an ongoing illness or injury that has lasting effects. 

In this case, the patient already has anemic syndrome with suspected chronic intoxication. Now, the aim is to prevent further exposure by changing his job. Primary prevention would have involved not working with the Xylene solvent in the first place.

 

All other options listed are geared towards treating the condition, however, the question is concerned about preventing further anemic complications which is to completely prevent exposure to the Xylene solvent.

189. A 49-year-old patient complains of itching, burning in the external genitals, frequent urination. The symptoms has been present for the last 7 months. The patient has irregular menstruation, once every 3-4 months. Over the last 2 years she presents with hot flashes, sweating, sleep disturbance. Examination revealed no pathological changes of the internal reproductive organs. Complete blood count and urinalysis showed no pathological changes. Vaginal smear contained 20- 25 leukocytes in the field of vision, mixed flora. What is the most likely diagnosis?

Explanation

This patient actually presented with Climacteric syndrome; one of the neuroendocrine syndromes in gynaecology that occurs during the transitional period ( starts before menopause); it occurs as a result of the gradual decrease in ovarian function and is characterised by; hot flashes, insomnia, nervousness, irregular menstruation , sleep disturbance etc. The term Climacteric syndrome is usually used interchangeably with Menopausal syndrome  but note that, Menopause refers to the complete cessation of menstruation for 12 straight months

 

The result from the examinations carried out  revealed no pathological  changes - this rules out the option of  cystitis, trichomonas colpitis, vulvitis and bacterial vaginosis.

190. A 56-year-old female patient complains of recurrent attacks of intensive pain irradiating along the ureters. Urine test results: protein - 0,37 g/l, RBCs- 20-25 in the field of vision, WBCs - 12- 14 in the field of vision. What method of instrumental diagnostics is the most informative for the diagnosis?

Explanation

 

Intravenous or Excretory Urography is an X-ray procedure that involves the intravenous administration of contrast material inorder to  verify and localize upper urinary tract diseases.  Retrograde pyelogram also uses a contrast agent for better visualization of the ureters and kidneys but note that: In intravenous pyelogram, the contrast dye is injected into  a vein while in retrograde pyelogram, it is injected directly into the ureters. Retrograde pyelogram is mostly done in cases where excretory urography does not give a clear image of the pathological area.

191. 2 weeks after labour a parturient woman developed breast pain being observed for 3 days. Examination revealed body temperature at the rate of 39oC, chills, weakness, hyperaemia, enlargement, pain and deformity of the mammary gland. On palpation the infiltrate was found to have an area of softening and fluctuation. What is the most likely diagnosis?

Explanation

Mastitis is inflammation of the breast tissue and can be broken down into lactational and non-lactational mastitis. Lactational mastitis is the most common form of mastitis. Lactational mastitis, also known as puerperal mastitis, is typically due to prolonged engorgement of milk ducts, with infectious components from the entry of bacteria through skin breaks. Patients can develop a focal area of erythema, pain, and swelling, and can have associated systemic symptoms, including fever. This occurs most commonly in the first six weeks of breastfeeding but can occur at any time during lactation, with most cases falling off after 3 months. Lactational mastitis is most commonly caused by bacteria that colonize the skin, with Staphylococcus aureus being the most common. Risk factors for lactational mastitis include prior history of mastitis, nipple cracks and fissures, inadequate milk drainage, maternal stress, lack of sleep, tight-fitting bras, and use of antifungal nipple creams.

 

From the question stem, we can differentiate the different types of mastitis listed. On palpation, an INFILTRATE was found with an area of softening and fluctuation (PURULENT). We can boldly conclude it is purulent because lactational/puerperal mastitis is commonly caused by bacteria which will produce a purulent inflammation.

192. A 22-day-old infant had developed red subcutaneous nodules from 1,0 to 1,5 cm large on the scalp. Later the nodules suppurated, body temperature rose up to 37, 7oC, there appeared symptoms of intoxication, the regional lymph nodes grew bigger. Blood test results: anemia, leukocytosis, neutrophilia, accelerated ESR. What is the most likely diagnosis?

Explanation

Pseudofurunculosis, is observed in neonates and infants. The disease begins with the appearance of superficial pustules in the ostium of sweat glands (periporitis). Caused by staphylococcus aureus, which penetrates in the depth of the sweat gland and causes the forming of deep indurated painful nodules. The nodules are very similar to furuncles, but they have not got core in the center (hence the name pseudofurunculosis).

 

Vesiculopustulosis is a widespread purulent disease, which appears in the first years of life. In the ostium of the sweat glands numerous pustules appear, filled with white yellow matter, the size of a pin head to a small pea, they do not merge with each other and are surrounded by bright edematous circles.

193. A 70-year-old patient consulted a doctor about arrhythmic cardiac activity, dyspnea. Objectively: AP- 150/90 mm Hg, extrasystole arrhythmia (10-12 beats per minute), left ventricular systolic dysfunction (ejection fraction at the rate of 42%). Which of antiarrhythmic drugs should be administered as initial therapy in this case?

Explanation

Amiodarone is a class III antiarrhythmic medication - potassium channel blockers. Amiodarone is a potent antiarrhythmic agent that is used to treat ventricular arrhythmias and atrial fibrillation. The drug prevents the recurrence of life-threatening ventricular arrhythmias and produces a modest reduction of sudden deaths in high-risk patients. Amiodarone is more effective than sotalol or propafenone in preventing recurrent atrial fibrillation in patients for whom a rhythm-control strategy is chosen.

 

Flecainide, Encainide, Moracizine are Class IC antiarrhythmic medication - Sodium channel blocker, commonly used only as a last resort in refractory Ventricular tachycardia. Digoxin is a cardiac glycoside that blocks Na+/K+ ATPase activity and improves contractility of the myocardium in heart failure. However, it can also be used in conditions such as Atrial Fibrillation.

194. A 38-year-old male works within the range of ionizing radiation. At a routine medical examination he presents no problems. In blood: RBCs - 4, 5·1012/l, Hb-80 g/l, WBCs - 2, 8 · 109/l, thrombocytes - 30 · 109/l. Decide if this person can work with sources of ionizing radiation:

Explanation

195. A puerpera breastfeeding for 1,5 weeks consulted a doctor about uniform breast engorgement. Breasts are painful. The body temperature is of 36, 6oC. Milk expressing is difficult. What is the most likely diagnosis?

Explanation

The key finding here is “Milk expressing is difficult” - this refers to a stagnation in the flow of milk in one or several ducts of the mammary gland leading to an enlarged and painful mammary gland; if not corrected, this leads to mastitis which is an inflammation (most often one-sided) of the mammary gland caused by pathogenic coccal flora (e.g. staphylococci).

This patient is yet to develop mastitis, however, if Lactostasis is not properly managed, it can result in Mastitis. Mastitis will be accompanied with fever, redness on the affected breast and pain. Of which this patient is afebrile and no redness stated in the question. 

It is important to treat blocked milk ducts so they do not progress to mastitis. Options include: 

-Making sure the baby is feeding well on the affected breast – offering the affected breast first can help. 

-The application of heat for a few minutes before a feed, gentle massage of the affected area during feeding, and cold packs after a feed and between feeds for comfort. 

-A change in feeding position. 

-Frequent drainage of the breast through feeding and expressing. 

 

If the blockage does not clear within 8 to 12 hours or you start to feel unwell, see your doctor.

196. A 28-year-old male patient complains of regurgitation, cough and heartburn that occurs every day after a meal, when bending forward or lying down. These problems have been observed for 4 years. Objective status and laboratory values are normal. FEGDS revealed endoesophagitis. What is the leading factor in the development of this disease?

Explanation

This patient presented with classic symptoms of Gastroesophageal Reflux Disease (GERD). Gastro-oesophageal reflux disease is a condition in which the reflux of gastric contents into the oesophagus provokes symptoms or complications and impairs quality of life. Typical symptoms of gastro-oesophageal reflux disease are heartburn and regurgitation. The pathogenesis of gastro-oesophageal reflux disease is multifactorial, involving transient lower esophageal sphincter relaxations (i.e failure of the inferior esophageal sphincter) and other lower esophageal sphincter pressure abnormalities. As a result, reflux of acid, bile, pepsin and pancreatic enzymes occurs, leading to esophageal mucosal injury as in this case Endoesophagitis. And the Esophagogastroduodenoscopy done confirms Endoesopahgitis as well.

H. pylori infection, Hypergastrinemia and Hypersecretion of hydrochloric acid can cause gastritis, gastric ulcer primarily but can also cause reflux disease if the lower or inferior esophageal sphincter is weak or relaxed. 

 

Duodeno-gastric reflux - Duodenum to Stomach.

197. A 24-year-old patient had been delivered to the thoracic department with a chest injury, a fracture of the IV, V, VI ribs on the right. Plan radiography shows the fluid level in the pleural cavity eaching the III rib on the right. Puncture blood contained clots. What is the optimal treatment tactics?

Explanation

Many patients with penetrating thoracic injuries do not survive to the Emergency Department (ED), and those that do survive to the ED often require immediate interventions such as tube thoracostomy, intubation, and thoracotomy. 

Thus, the emergency clinician must be able to rapidly diagnose and treat injuries that are immediately life-threatening as in this case that presented with a Gunshot injury.

The above patient presents with a major trauma to the right side of the chest which is confirmed by a hemothorax (Hemothorax). 

Thoracotomy is a surgical procedure carried out inorder to have access to thoracic organs such as the lungs, heart or esophagus. This procedure is the first step in gaining access to the thoracic cavity and thus is indicated in cases of a lobectomy, major trauma of the chest, pneumonectomy. In the case of this patient, an emergency thoracotomy should be carried out in order to drain the blood and prevent compression and eventual collapse of the lungs.

Thoracentesis or Pleural puncture is a procedure in which a needle is inserted into the pleural space between the lungs and the chest wall. This procedure is done to remove excess fluid, known as a pleural effusion, from the pleural space to help you breathe easier. In this case, it’s not just fluid, there are blood clots. So, an incision (thoracotomy) should be made and arrest the bleeding, then drain. This patient needs this to be done as soon as possible, that’s why the best answer choice is the Emergency Thoracotomy.

 

It’s not enough to use hemostatic therapy, the clots must be removed and blood drained. Thoracoscopy is an imaging technique and the X-ray has given enough information at this point.

198. A 10-year-old patient complains of skin itch that occurs at night. Objectively: multiple paired papules, burrow tracks in the interdigital skin folds, on the anterolateral surfaces of abdomen and buttocks. Specify the period of regular medical check-up for pupils of the class where the patient learns:

Explanation

Scabies is an itchy skin condition caused by a tiny burrowing mite called Sarcoptes scabiei. Intense itching occurs in the area where the mite burrows. The urge to scratch may be especially strong at night. The first time a person is exposed to the scabies mite, it can take upwards of 2 to 6 weeks for symptoms to develop. So, suspected contacts i.e. pupils of the class where the patient learns can be checked in 6 - 8 weeks (approximately 2 months) to see if they’ve manifested any symptom to the infection.

Scabies is contagious and can spread quickly through close physical contact in a family, child care group, school class, nursing home or prison. Because scabies is so contagious, doctors often recommend treatment for entire families or contact groups.

The most common site of infestation in adults and older children include: in between the fingers; around fingernails; armpits; waistline; inner parts of the wrists; inner elbow; soles of the feet; the breasts, particularly the areas around the nipples; male genitalia; buttocks; knees; shoulder blades.

 

Infants and young children experience infestation in other areas of the body, including the: scalp; face; neck; palms of the hands; soles of the feet.

199. Examination of a Rh-negative pregnant woman at 32 weeks of gestation revealed a four-time rise of Rh-antibody titer within 2 weeks, the titer was 1:64. In the first two pregnancies the patient had experienced antenatal fetal death due to hemolytic disease. What is the optimal tactics of pregnancy management?

Explanation

This question is talking about Rh incompatibility. This occurs when a Rhesus negative (Rh-) mother marries a Rh+ Man and they conceive a Rh+ child. Note: the mother must be Rh- and the child Rh+. The first child with Rh+ usually survives, but subsequently Rh+ fetus will be attacked by Rh antibodies in the mother which crosses the placenta to attack the fetal red blood cells causing hemolysis.

From the question, the patient already had 2 antenatal fetal deaths due to hemolytic disease. And in this pregnancy, it is observed that the Rh antibodies are beginning to rise (4 fold increase), therefore, another hemolytic disease is imminent.

The first Rh+ child usually survives because the mother is yet to develop the Rh antibodies. Usually, the Rh- mother will be exposed to these antibodies when the Rh+ antigens from the first child cross into her blood during delivery, or CS. In that case, the mother will develop Rh antibodies for any subsequent pregnancy that is Rh+.

Once this occurs, i.e. once a Rh- mother conceives a Rh+ child, she should receive RhoGAM at 28weeks of gestation and within 72hours after delivery. This will prevent the development of antibodies. However, if she did not receive the RhoGAM, she will develop the antibodies just as it is seen in this case and the best option is to deliver the fetus as soon as possible to prevent hemolytic reactions.

She’s already at 32weeks of gestation  and if she took RhoGAM @28weeks, the antibodies will not rise. It’s medically wrong to leave the pregnancy till 37 weeks because there is a high chance the antibodies must have risen to a considerable level to cause massive hemolysis in the developing fetus before getting to 37 weeks of gestation. Ultrasound would not have any impact in this case.

There is no need waiting for another 2 weeks to screen for Rh antibodies. From the history, there are 2 confirmed cases of hemolytic disease in the past, she’s Rh- and the Rh antibodies are already increasing. 

Anti-Rh immunoglobulin is effective when given to a Rh- mother @ 26-28 weeks of gestation and within 72hours after delivery of the first child that is Rh+. 

 

The best management is to deliver the child while the fetus is still alive and hemolytic reaction is yet to start. At 32weeks, the mother should receive Corticosteroids to help the fetal lungs develop and deliver the fetus as soon as possible.

200. A 7-year-old female child has developed an acute condition. She complains of a headache, two onsets of vomiting. Objectively: deferred reactions, body temperature - 39, 3oC, pronounced hyperesthesia, nuchal rigidity, positive superior and inferior Brudzinski’s signs, symmetric Kernig’s sign. What is the provisional diagnosis?

Explanation

Meningitis is a common life-threatening medical emergency caused by infectious and non-infectious agents. It is a life-threatening medical emergency characterized by the inflammation of the membranes surrounding your spinal cord and brain. Kernig's sign, Brudzinski's sign, and nuchal rigidity are bedside diagnostic signs used to evaluate suspected cases of meningitis.

To elicit the Kernig's sign, the patient is kept in supine position, hip and knee are flexed to a right angle, and then knee is slowly extended by the examiner. The appearance of resistance or pain during extension of the patient's knees beyond 135 degrees constitutes a positive Kernig's sign. Another way to explain this:

To look for Kernig’s sign:

  1. Lie face up.

  2. Flex your knee and hip in a 90˚ angle while someone else slowly extends your knee.

If you feel either resistance or pain, see a doctor right away for treatment.

To check for the Brudzinski sign:

  1. Lie flat on your back.

  2. Your doctor will place one hand behind your head, and another on your chest to prevent you from rising.

  3. Then, your doctor will lift your head, bringing your chin to your chest.

A positive Brudzinski sign occurs when this causes flexion of the hips.

 

A third sign used to diagnose meningitis is called nuchal rigidity. Nuchal rigidity is an inability to flex the neck forward due to rigidity of the neck muscles.